Exam 6 NCLEX Questions

अब Quizwiz के साथ अपने होमवर्क और परीक्षाओं को एस करें!

The primary health care provider is planning to administer a skeletal muscle relaxant to a client with a spinal cord injury. The medication will be administered intrathecally. Which medication should the nurse expect to be prescribed and administered by this route? 1. Baclofen 2. Chlorzoxazone 3. Dantrolene sodium 4. Cyclobenzaprine hydrochloride

1. Baclofen Rationale: Baclofen is the skeletal muscle relaxant that can be administered intrathecally, which means within the spinal column. Therefore, the remaining options are incorrect.

The nurse is caring for a client with a head injury. The client's intracranial pressure reading is 8 mm Hg. Which condition should the nurse document? 1. The intracranial pressure reading is normal. 2. The intracranial pressure reading is elevated. 3. The intracranial pressure reading is borderline. 4. An intracranial pressure reading of 8 mm Hg is low.

1. The intracranial pressure reading is normal. Rationale: The normal intracranial pressure is 5 to 15 mm Hg. A pressure of 8 mm Hg is within normal range.

A client with a diagnosis of trigeminal neuralgia is started on a regimen of carbamazepine. The nurse provides instructions to the client about the side and adverse effects of the medication. Which client statement indicates an understanding of the side and adverse effects of the medication? 1. "I will report a fever or sore throat to my primary health care provider." 2. "I must brush my teeth frequently to avoid damage to my gums." 3. "If I notice ringing in my ears that doesn't stop, I'll seek medical attention." 4. "If I notice a pink color to my urine, I will stop the medication and call my health care provider."

1. "I will report a fever or sore throat to my primary health care provider." Rationale: Agranulocytosis is an adverse effect of carbamazepine and places the client at risk for infection. If a fever or a sore throat develops, the primary health care provider should be notified. Gum damage, ringing in the ears, and pink-colored urine are not effects associated with this medication.

A neurologist prescribed ticlopidine to the client with thrombotic stroke. The nurse provides instructions to the client and spouse regarding the medication. Which statement made by the client indicates that education was effective? 1. "I'll take the medicine with meals." 2. "If I do not feel well, I should skip the medication." 3. "I won't have another stroke if I take this medicine faithfully." 4. "If I have any gastrointestinal side effects, I should call the neurologist."

1. "I'll take the medicine with meals." Rationale: Ticlopidine is an antiplatelet agent that is used to assist in preventing a thrombotic stroke. Ticlopidine is best tolerated when taken with meals. The most common side effects are gastrointestinal (GI) disturbances. Taking ticlopidine with meals tends to lessen those effects. It is not necessary to contact the neurologist or prescribing provider if GI upset occurs. The client should not skip medications. The medication is used to prevent strokes but does not guarantee that a stroke will not occur.

The nurse is caring for a client with myasthenia gravis who has received edrophonium by the intravenous route to test for myasthenic crisis. The client asks the nurse how long the improvement in muscle strength will last. Which response should the nurse make to the client? 1. "It will last for 4 to 5 minutes." 2. "It will last for about 30 minutes." 3. "It will last longer than 60 minutes." 4. "It will last approximately 10 minutes."

1. "It will last for 4 to 5 minutes." Rationale: Edrophonium commonly is given to test for myasthenic crisis. If the client is in myasthenic crisis, muscle strength improves after administration of the medication. Within 30 to 60 seconds, most myasthenic clients show a marked improvement in muscle tone that lasts for 4 to 5 minutes. Options 2, 3, and 4 are incorrect.

A client with a history of spinal cord injury is beginning medication therapy with baclofen. The nurse determines that the client understands the side/adverse effects of the medication if the client makes which statement? 1. "The medication may make me drowsy." 2. "The medication can cause high blood pressure." 3. "The medication may cause me to have some muscle pain." 4. "The medication may increase my sensitivity to bright light."

1. "The medication may make me drowsy." Rationale: Baclofen is a central-acting skeletal muscle relaxant useful in treating muscle spasticity, usually in upper motor neuron injury. Side/adverse effects include drowsiness, dizziness, weakness, and nausea. Occasional side effects include headache, paresthesias of the hands and feet, constipation or diarrhea, anorexia, hypotension, confusion, and nasal congestion. The other options are incorrect.

A client with recent-onset Bell's palsy is upset and crying about the change in facial appearance. The nurse plans to support the client emotionally by making which statement to the client? 1. "This is not a stroke, and many clients recover in 3 to 5 weeks." 2. "This is caused by a small tumor, which can be removed easily." 3. "This is similar to a stroke, but all symptoms will reverse without treatment." 4. "This is a temporary problem, with treatment similar to that for migraine headaches."

1. "This is not a stroke, and many clients recover in 3 to 5 weeks." Rationale: Clients with Bell's palsy should be reassured that they have not experienced a stroke (brain attack) and that symptoms often disappear spontaneously in 3 to 5 weeks. The client is given supportive treatment for symptoms. Bell's palsy usually is not caused by a tumor, and the treatment is not similar to that for migraine headaches.

The nurse is reviewing the laboratory results of a serum medication level assay for a client seen in the health care clinic who has been taking phenytoin for the control of seizures. The nurse determines that a subtherapeutic level of phenytoin is present and that additional medication is required if which level is found? 1. 3 mcg/mL (12 mcmol/L) 2. 16 mcg/mL (63 mcmol/L) 3. 18 mcg/mL (71 mcmol/L) 4. 24 mcg/mL (95 mcmol/L)

1. 3 mcg/mL (12 mcmol/L) Rationale: The therapeutic range for a serum phenytoin level is 10 to 20 mcg/mL (40 to 79 mcmol/L). A level of 3 mcg/dL (12 mcmol/L) is subtherapeutic and would indicate the need for additional medication. If the level is less than the therapeutic range, the client may continue to experience seizure activity. The level in option 4 is high. If the level is too high, the client could experience phenytoin toxicity.

A client with status epilepticus has been prescribed phenytoin to be given by the intravenous (IV) route. The nurse administering the medication is careful not to exceed which recommended infusion rate? 1. 50 mg/min 2. 60 mg/min 3. 100 mg/min 4. 750 mg/min

1. 50 mg/min Rationale: IV administration of phenytoin is performed slowly (no faster than 50 mg/min) because rapid administration can cause cardiovascular collapse. It should not be added to any existing IV infusion because this is likely to produce a precipitate in the solution. Solutions are highly alkaline and can cause local venous irritation.

A client with a spinal cord injury at the level of C5 has a weakened respiratory effort and ineffective cough and is using accessory neck muscles in breathing. The nurse carefully monitors the client and suspects the presence of which problem? 1. Altered breathing pattern 2. Increased likelihood of injury 3. Ineffective oxygen consumption 4 .Increased susceptibility to aspiration

1. Altered breathing pattern Rationale: Altered breathing pattern indicates that the respiratory rate, depth, rhythm, timing, or chest wall movements are insufficient for optimal ventilation of the client. This is a risk for clients with spinal cord injury in the lower cervical area. Ineffective oxygen consumption occurs when oxygenation or carbon dioxide elimination is altered at the alveolar-capillary membrane. Increased susceptibility to aspiration and increased likelihood of injury are unrelated to the subject of the question.

The nurse is caring for a client diagnosed with bacterial meningitis. Which clinical manifestation should the nurse monitor for, indicating increased intracranial pressure? 1. Altered mental status 2. Decreased urinary output 3. Decreased peripheral sensation 4. Numbness and tingling in the fingers and toes

1. Altered mental status Rationale: Meningitis is a bacterial infection of the meninges of the brain. A common complication of meningitis is increased intracranial pressure. Altered mental status can result from increased intracranial pressure. Decreased urinary output, decreased peripheral sensation, and numbness and tingling in the fingers and toes are not specifically associated with bacterial meningitis.

The nurse is caring for a client who is in the chronic phase of stroke (brain attack) and has a right-sided hemiparesis. The nurse identifies that the client is unable to feed self. Which is the appropriate nursing intervention? 1. Assist the client to eat with the left hand to build strength. 2. Provide a pureed diet that is easy for the client to swallow. 3. Inform the client that a feeding tube will be placed if progress is not made. 4. Provide a variety of foods on the meal tray to stimulate the client's appetite.

1. Assist the client to eat with the left hand to build strength. Rationale: Right-sided hemiparesis is weakness of the right arm and leg. The nurse should teach the client to use both sides of the body to increase strength and build endurance. Providing a pureed diet is incorrect. The question does not mention swallowing difficulty, so there is no need to puree the food. Informing the client that a feeding tube may need to be placed is incorrect. That information would come from the primary health care provider. Providing a variety of foods is also incorrect because the problem is not the food selection but the client's ability to eat the food independently

A client with myasthenia gravis arrives at the hospital emergency department in suspected crisis. The primary health care provider plans to administer edrophonium to differentiate between myasthenic and cholinergic crises. The nurse ensures that which medication is available in the event that the client is in cholinergic crisis? 1. Atropine sulfate 2. Morphine sulfate 3. Protamine sulfate 4. Pyridostigmine bromide

1. Atropine sulfate Rationale: Clients with cholinergic crisis have experienced an overdosage of medication. Edrophonium will exacerbate symptoms in cholinergic crisis to the point at which the client may need intubation and mechanical ventilation. Intravenous atropine sulfate is used to reverse the effects of these anticholinesterase medications. Morphine sulfate and pyridostigmine bromide would worsen the symptoms of cholinergic crisis. Protamine sulfate is the antidote for heparin.

The nurse in the neurological unit is monitoring a client with a head injury for signs of increased intracranial pressure (ICP). The nurse reviews the assessment findings for the client and notes documentation of the presence of Cushing's reflex. The nurse determines that the presence of this reflex is obtained by assessing which item? 1. Blood pressure 2. Motor response 3. Pupillary response 4. Level of consciousness

1. Blood pressure Rationale: Cushing's reflex is a late sign of increased ICP and consists of a widening pulse pressure (systolic pressure rises faster than diastolic pressure) and bradycardia. The remaining options are unrelated to monitoring for Cushing's reflex.

A client is admitted to the hospital with a diagnosis of neurogenic shock after a traumatic motor vehicle collision. Which manifestation best characterizes this diagnosis? 1. Bradycardia 2. Hyperthermia 3. Hypoglycemia 4. Increased cardiac output

1. Bradycardia Rationale: Neurogenic shock can occur after a spinal cord injury. Usually the body attempts to compensate massive vasodilation by becoming tachycardic to increase the amount of blood flow and oxygen delivered to the tissues; however, in neurogenic shock, the sympathetic nervous system is disrupted, so the parasympathetic system takes over, resulting in bradycardia. This insufficient pumping of the heart leads to a decrease in cardiac output. Hypoglycemia is not an indicator of neurogenic shock. Hypothermia develops because of the vasodilation and the inability to control body temperature through vasoconstriction.

The home health nurse is visiting a client with myasthenia gravis and is discussing methods to minimize the risk of aspiration during meals related to decreased muscle strength. Which suggestions should the nurse give to the client? Select all that apply. 1. Chew food thoroughly. 2. Cut food into very small pieces. 3. Sit straight up in the chair while eating. 4. Lift the head while swallowing liquids. 5. Swallow when the chin is tipped slightly downward to the chest

1. Chew food thoroughly. 2. Cut food into very small pieces. 3. Sit straight up in the chair while eating. 5. Swallow when the chin is tipped slightly downward to the chest Rationale: The client avoids swallowing any type of food or drink with the head lifted upward, which could actually cause aspiration by opening the glottis. The client should be advised to sit upright while eating, not to talk with food in the mouth (talking requires opening the glottis), cut food into very small pieces, chew thoroughly, and tip the chin downward to swallow.

The nurse reviews the primary health care provider's (PHCP's) prescriptions for a client with Guillain-Barré syndrome. Which prescription written by the PHCP should the nurse question? 1. Clear liquid diet 2. Bilateral calf measure 3. Monitor vital signs frequently 4. Passive range-of-motion (ROM) exercises

1. Clear liquid diet Rationale: Clients with Guillain-Barré syndrome have dysphagia. Clients with dysphagia are more likely to aspirate clear liquids than thick or semisolid foods. Passive ROM exercises can help prevent contractures, and assessing calf measurements can help detect deep vein thrombosis, for which these clients are at risk. Because clients with Guillain-Barré syndrome are at risk for hypotension or hypertension, bradycardia, and respiratory depression, frequent monitoring of vital signs is required.

The nurse is caring for a client who sustained a spinal cord injury. During administration of morning care, the client begins to exhibit signs and symptoms of autonomic dysreflexia. Which initial nursing action should the nurse take? 1. Elevate the head of the bed. 2. Examine the rectum digitally. 3. Assess the client's blood pressure. 4. Place the client in the prone position.

1. Elevate the head of the bed. Rationale: Autonomic dysreflexia is a serious complication that can occur in the spinal cord-injured client. Once the syndrome is identified, the nurse elevates the head of the client's bed and then examines the client for the source of noxious stimuli. The nurse also assesses the client's blood pressure, but the initial action would be to elevate the head of the bed. The client would not be placed in the prone position; lying flat will increase the client's blood pressure.

The nurse is caring for a client who is on bed rest as part of aneurysm precautions. The nurse should avoid doing which action when giving respiratory care to this client? 1. Encouraging hourly coughing 2. Assisting with incentive spirometer 3. Encouraging hourly deep breathing 4. Repositioning gently side to side every 2 hours

1. Encouraging hourly coughing Rationale: With aneurysm precautions, any activity that could raise the client's intracranial pressure (ICP) is avoided. For this reason, activities such as straining, coughing, blowing the nose, and even sneezing are avoided whenever possible. The other interventions (repositioning, deep breathing, and incentive spirometry) do not provide added risk of increasing ICP and are beneficial in reducing the respiratory complications of bed rest.

The nurse is performing an assessment on a client with a diagnosis of Bell's palsy. The nurse should expect to observe which finding in the client? 1. Facial drooping 2. Periorbital edema 3. Ptosis of the eyelid 4. Twitching on the affected side of the face

1. Facial drooping Rationale: Bell's palsy is a one-sided facial paralysis caused by the compression of the facial nerve (cranial nerve VII). Assessment findings include facial droop from paralysis of the facial muscles; increased lacrimation; painful sensations in the eye, face, or behind the ear; and speech or chewing difficulty. The remaining options are not associated findings in Bell's palsy.

The nurse has just admitted to the nursing unit a client with a basilar skull fracture who is at risk for increased intracranial pressure. Pending specific primary health care provider prescriptions, the nurse should safely place the client in which positions? Select all that apply. 1. Head midline 2. Neck in neutral position 3. Head of bed elevated 30 to 45 degrees 4. Head turned to the side when flat in bed 5. Neck and jaw flexed forward when opening the mouth

1. Head midline 2. Neck in neutral position 3. Head of bed elevated 30 to 45 degrees Rationale: Use of proper positions promotes venous drainage from the cranium to keep intracranial pressure from elevating. The head of the client at risk for or with increased intracranial pressure should be positioned so that it is in a neutral, midline position. The head of the bed should be raised to 30 to 45 degrees. The nurse should avoid flexing or extending the client's neck or turning the client's head from side to side.

The nurse has just admitted to the nursing unit a client with a basilar skull fracture who is at risk for increased intracranial pressure (ICP). Pending specific primary health care provider prescriptions, the nurse should plan to place the client in which positions? Select all that apply. 1. Head midline 2. Neck in neutral position 3. Flat, with head turned to the side 4. Head of bed elevated 30 to 45 degrees 5. Head of bed elevated with the neck extended

1. Head midline 2. Neck in neutral position 4. Head of bed elevated 30 to 45 degrees Rationale: The client who is at risk for or who has increased ICP should be positioned so that the head is in a neutral, midline position. The nurse should avoid flexing or extending the client's neck or turning the head from side to side. The head of the bed should be raised to 30 to 45 degrees. Use of proper positions promotes venous drainage from the cranium to keep ICP down.

An adolescent who has a gunshot wound to the head is being admitted to the critical care unit from the emergency department. Which of the following assessment findings are indicative of increased ICP? Select all that apply. 1. Headache 2. Dilated pupils 3. Tachycardia 4. Decorticate posturing 5. Hypotension

1. Headache 2. Dilated pupils 4. Decorticate posturing Rationale: HA, dilated pupils and decorticate/decerebrate posturing are signs of ICP. As are bradycardia and hypertension

A client with a spinal cord injury is prone to experiencing autonomic dysreflexia. The nurse should include which measures in the plan of care to minimize the risk of occurrence? Select all that apply. 1. Keeping the linens wrinkle-free under the client 2. Preventing unnecessary pressure on the lower limbs 3. Limiting bladder catheterization to once every 12 hours 4. Turning and repositioning the client at least every 2 hours 5. Ensuring that the client has a bowel movement at least once a week

1. Keeping the linens wrinkle-free under the client 2. Preventing unnecessary pressure on the lower limbs 4. Turning and repositioning the client at least every 2 hours Rationale: The most frequent cause of autonomic dysreflexia is a distended bladder. Straight catheterization should be done every 4 to 6 hours (catheterization every 12 hours is too infrequent), and urinary catheters should be checked frequently to prevent kinks in the tubing. Constipation and fecal impaction are other causes, so maintaining bowel regularity is important. Ensuring a bowel movement once a week is much too infrequent. Other causes include stimulation of the skin from tactile, thermal, or painful stimuli. The nurse administers care to minimize risk in these areas.

A client experiencing spasticity as a result of spinal cord injury has a new prescription for dantrolene. Before administering the first dose, the nurse checks to see if which baseline study has been done? 1. Liver function studies 2. Renal function studies 3. Otoscopic examination 4. Blood glucose measurements

1. Liver function studies Rationale: Dantrolene is a skeletal muscle relaxant and can cause liver damage; therefore, the nurse should monitor the results of liver function studies. They should be done before therapy starts and periodically throughout therapy. Dantrolene is discontinued if no relief of spasticity is achieved in 6 weeks. The incorrect options are not specifically related to the administration of this medication.

The nurse is caring for a client who begins to experience seizure activity while in bed. Which actions should the nurse take? Select all that apply. 1. Loosening restrictive clothing. 2. Restraining the client's limbs. 3. Removing the pillow and raising padded side rails. 4. Positioning the client to the side, if possible, with the head flexed forward. 5. Keeping the curtain around the client and the room door open so when help arrives they can quickly enter to assist.

1. Loosening restrictive clothing. 3. Removing the pillow and raising padded side rails. 4. Positioning the client to the side, if possible, with the head flexed forward. Rationale: Nursing actions during a seizure include providing for privacy, loosening restrictive clothing, removing the pillow and raising padded side rails in the bed, and placing the client on 1 side with the head flexed forward, if possible, to allow the tongue to fall forward and facilitate drainage. The limbs are never restrained because the strong muscle contractions could cause the client harm. If the client is not in bed when seizure activity begins, the nurse lowers the client to the floor, if possible; protects the head from injury; and moves furniture that may injure the client.

The nurse is caring for a client with an intracranial aneurysm who has been alert. Which signs and symptoms are an early indication that the level of consciousness (LOC) is deteriorating? Select all that apply. 1. Mild drowsiness 2. Drooping eyelids 3. Ptosis of the left eyelid 4. Slight slurring of speech 5. Less frequent spontaneous speech

1. Mild drowsiness 4. Slight slurring of speech 5. Less frequent spontaneous speech Rationale: Early changes in LOC relate to orientation, alertness, and verbal responsiveness. Mild drowsiness, slight slurring of speech, and less frequent spontaneous speech are early signs of decreasing LOC. Ptosis (drooping) of the eyelid is caused by pressure on and dysfunction of cranial nerve III. Once ptosis occurs, it is ongoing; it does not relate to LOC.

A thymectomy accomplished via a median sternotomy approach is performed in a client with a diagnosis of myasthenia gravis. The nurse creates a postoperative plan of care for the client that should include which intervention? 1. Monitor the chest tube drainage. 2. Restrict visitors for 24 hours postoperatively. 3. Maintain intravenous infusion of lactated Ringer's solution. 4. Avoid administering pain medication to prevent respiratory depression.

1. Monitor the chest tube drainage. Rationale: The thymus has played a role in the development of myasthenia gravis. A thymectomy is the surgical removal of the thymus gland and may be used for management of clients with myasthenia gravis to improve weakness. The procedure is performed through a median sternotomy or a transcervical approach. Postoperatively the client will have a chest tube in the mediastinum. Lactated intravenous solutions usually are avoided because they can increase weakness. Pain medication is administered as needed, but the client is monitored closely for respiratory depression. There is no reason to restrict visitors.

Which interventions would be included in the care of a client with a head injury and a subarachnoid bolt? Select all that apply. 1. Monitor vital signs. 2. Monitor neurological status. 3. Monitor the dressing for signs of infection. 4. Monitor for signs of increased intracranial pressure. 5. Drain cerebrospinal fluid when the intracranial pressure is elevated.

1. Monitor vital signs. 2. Monitor neurological status. 3. Monitor the dressing for signs of infection. 4. Monitor for signs of increased intracranial pressure. Rationale: A subarachnoid bolt is inserted into the subarachnoid space and is used to measure intracranial pressure. Because a subarachnoid bolt is placed in the subarachnoid space, it is not capable of draining cerebrospinal fluid, which is produced in the ventricles. Therefore, the option to drain cerebrospinal fluid is not an intervention. The remaining options are appropriate interventions.

A client with multiple sclerosis tells a home health care nurse that she is having increasing difficulty in transferring from the bed to a chair. What is the initial nursing action? 1. Observe the client demonstrating the transfer technique. 2. Start a restorative nursing program before an injury occurs. 3. Seize the opportunity to discuss potential nursing home placement. 4. Determine the number of falls that the client has had in recent weeks.

1. Observe the client demonstrating the transfer technique. Rationale: Observation of the client's transfer technique is the initial intervention. Starting a restorative program is important but not unless an assessment has been completed first. Discussing nursing home placement would be inappropriate in view of the information provided in the question. Determining the number of falls is another important intervention, but observing the transfer technique should be done first.

The home care nurse is visiting a male client who is recovering at home after suffering a brain attack (stroke) 2 weeks ago. The client's wife states that the client has difficulty feeding himself and difficulty with swallowing food and fluids. Which would be the initial nursing action? 1. Observe the client feeding himself. 2. Observe the wife feeding the client. 3. Arrange for a home health aide to assist at mealtimes. 4. Instruct the wife in the use of a feeding syringe to feed the client.

1. Observe the client feeding himself. Rationale: It is not uncommon for a client to have difficulty swallowing after experiencing a stroke. Often the client has hemiplegia. The arm on the affected side may be paralyzed, and the client may have to learn to use the opposite arm for self-feeding. Using the nondominant arm may require rehabilitation and retraining. Also, a client may have partial paralysis of the mouth, tongue, or esophagus. To best assist the client, the nurse should first assess the situation by watching the self-feeding process. Perhaps the problem lies in the feeding technique, the type of feeding tool used, the types of foods being served, or a combination. Having someone else feed the client may be necessary if self-feeding is not possible. This approach, however, does not promote independence for the client. A feeding syringe is not recommended for feeding most clients.

A client with myasthenia gravis who is taking neostigmine is experiencing frequent exacerbations of myasthenic crisis and cholinergic crisis. The nurse teaches the client that it is most important that this medication be taken in which manner? 1. On time 2. On an empty stomach 3. Double-dosed if 1 dose is missed 4. Titrated for dosage, depending on symptoms

1. On time Rationale: The client should take neostigmine exactly on time. Taking the medication early or late could result in myasthenic or cholinergic crisis. Taking the medication on time is especially important for the client with dysphagia because the client may not be able to swallow the medication if it is given late. These clients are taught to set an alarm clock to remind them of dosage times. The medication should be administered with food or milk to minimize side and adverse effects. The client should never skip or double up on missed doses or titrate the dose, depending on symptoms. The client needs to take the medication exactly as prescribed.

The nurse is instituting seizure precautions for a client who is being admitted from the emergency department. Which measures should the nurse include in planning for the client's safety? Select all that apply. 1. Padding the side rails of the bed. 2. Placing an airway at the bedside. 3. Placing the bed in the high position. 4. Putting a padded tongue blade at the head of the bed. 5. Placing oxygen and suction equipment at the bedside. 6. Flushing the intravenous catheter to ensure that the site is patent.

1. Padding the side rails of the bed. 2. Placing an airway at the bedside. 5. Placing oxygen and suction equipment at the bedside. 6. Flushing the intravenous catheter to ensure that the site is patent. Rationale: Seizure precautions may vary from agency to agency, but they generally have some common features. Usually, an airway, oxygen, and suctioning equipment are kept available at the bedside. The side rails of the bed are padded, and the bed is kept in the lowest position. The client has an intravenous access in place to have a readily accessible route if antiseizure medications must be administered, and as part of the routine assessment the nurse should be checking patency of the catheter. The use of padded tongue blades is highly controversial, and they should not be kept at the bedside. Forcing a tongue blade into the mouth during a seizure more likely will harm the client who bites down during seizure activity. Risks include blocking the airway from improper placement, chipping the client's teeth, and subsequent risk of aspirating tooth fragments. If the client has an aura before the seizure, it may give the nurse enough time to place an oral airway before seizure activity begins.

The nurse is caring for a client who has suffered a spinal cord injury. The nurse further assesses the client for other signs of autonomic dysreflexia if the client experiences which of the following? Select all that apply. 1. Severe throbbing headache 2. Pallor of the face and neck 3. Blurred vision 4. Hypertension 5. Tachycardia

1. Severe throbbing headache 3. Blurred vision 4. Hypertension Rationale: Patient will have flushing of skin above level of injury, will be hypertensive, with bradycardia, and have a severe throbbing headache.

The nurse is assessing a client who is experiencing seizure activity. The nurse understands that it is necessary to determine information about which items as part of routine assessment of seizures? Select all that apply. 1. Postictal status 2. Duration of the seizure 3. Changes in pupil size or eye deviation 4. Seizure progression and type of movements 5. What the client ate in the 2 hours preceding seizure activity

1. Postictal status 2. Duration of the seizure 3. Changes in pupil size or eye deviation 4. Seizure progression and type of movements Rationale: Typically seizure assessment includes the time the seizure began, parts of the body affected, type of movements and progression of the seizure, change in pupil size or eye deviation or nystagmus, client condition during the seizure, and postictal status. Determining what the client ate 2 hours prior to the seizure is not a component of seizure assessment.

The nurse is preparing a plan of care for a client with a diagnosis of amyotrophic lateral sclerosis (ALS). On assessment, the nurse notes that the client is severely dysphagic. Which intervention should be included in the care plan for this client? Select all that apply. 1. Provide oral hygiene after each meal. 2. Assess swallowing ability frequently. 3. Allow the client sufficient time to eat. 4. Maintain a suction machine at the bedside. 5. Provide a full liquid diet for ease in swallowing.

1. Provide oral hygiene after each meal. 2. Assess swallowing ability frequently. 3. Allow the client sufficient time to eat. 4. Maintain a suction machine at the bedside. Rationale: A client who is severely dysphagic is at risk for aspiration. Swallowing is assessed frequently. The client should be given a sufficient amount of time to eat. Semisoft foods are easiest to swallow and require less chewing. Oral hygiene is necessary after each meal. Suctioning should be available for clients who experience dysphagia and are at risk for aspiration.

The nurse is planning to put aneurysm precautions in place for a client with a cerebral aneurysm. Which nursing measures would be implemented? Select all that apply. 1. Provide physical aspects of care. 2. Prevent pushing or straining activities. 3. Limit caffeinated coffee to 1 cup per day. 4. Keeping the lights on in the client's room. 5. Maintain the head of the bed at 15 degrees.

1. Provide physical aspects of care. 2. Prevent pushing or straining activities. 5. Maintain the head of the bed at 15 degrees. Rationale: Aneurysm precautions include placing the client on bed rest (as prescribed) in a quiet setting. Stimulants such as caffeine and nicotine are prohibited; decaffeinated coffee or tea may be used. Lights are kept dim to minimize environmental stimulation. Any activity that increases the blood pressure or impedes venous return from the brain is prohibited, such as pushing, pulling, sneezing, coughing, or straining. The nurse provides physical care to minimize increases in blood pressure. For the same reason, visitors, radio, television, and reading materials are prohibited or limited.

A client who has experienced a stroke has partial hemiplegia of the left leg. The nurse interprets that the client could benefit from the support and stability provided by which item? 1. Quad cane 2. Wheelchair 3. Lofstrand crutch 4. Aluminum crutch

1. Quad cane Rationale: A quad cane may be used by the client requiring greater support and stability than is provided by a straight leg cane. The quad cane provides a four-point base of support and is indicated for use by clients with partial or complete hemiplegia. Neither crutches nor a wheelchair is indicated for use with a client such as the one described in the question.

The client with a head injury is experiencing signs of increased intracranial pressure (ICP), and mannitol is prescribed. The nurse administering this medication expects which as intended effects of this medication? Select all that apply. 1. Reduced ICP 2. Increased diuresis 3. Increased osmotic pressure of glomerular filtrate 4. Reduced tubular reabsorption of water and solutes 5. Reabsorption of sodium and water in the loop of Henle

1. Reduced ICP 2. Increased diuresis 3. Increased osmotic pressure of glomerular filtrate 4. Reduced tubular reabsorption of water and solutes Rationale: Mannitol is an osmotic diuretic that induces diuresis by raising the osmotic pressure of glomerular filtrate, thereby inhibiting tubular reabsorption of water and solutes. It is used to reduce intracranial pressure in the client with head trauma. The incorrect option would cause fluid retention through reabsorption, thereby increasing ICP.

A registered nurse (RN) asks a licensed practical nurse (LPN) to set up a hospital room for a client who is being admitted with a diagnosis of tonic-clonic seizures and asks the LPN to institute seizure precautions. The RN checks the client's room before the arrival of the client and determines that which item placed in the room by the LPN is unsafe? 1. Restraints 2. Nasal cannula 3. Suction catheter 4. Padding for side rails

1. Restraints Rationale: Seizure precautions include keeping side rails up and padded if the client has tonic-clonic seizures, ensuring that suction and oxygen equipment is available, and disabling the locks on the bathroom and room doors. Restraints are not used and can result in client injury.

The nurse is trying to communicate with a client who had a stroke and has aphasia. Which actions by the nurse would be most helpful to the client? Select all that apply. 1. Speaking to the client at a slower rate 2. Allowing plenty of time for the client to respond 3. Completing the sentences that the client cannot finish 4. Looking directly at the client during attempts at speech 5. Shouting words if it seems as though the client has difficulty understanding

1. Speaking to the client at a slower rate 2. Allowing plenty of time for the client to respond 4. Looking directly at the client during attempts at speech Rationale: Clients with aphasia after brain attack often fatigue easily and have a short attention span. General guidelines when trying to communicate with the aphasic client include speaking more slowly and allowing adequate response time, listening to and watching attempts to communicate, and trying to put the client at ease with a caring and understanding manner. The nurse would avoid shouting (because the client is not deaf), appearing rushed for a response, and letting family members provide all responses for the client.

A client has suffered damage to Broca's area of the brain. Which priority assessment should the nurse perform? 1. Speech 2. Hearing 3. Balance 4. Level of consciousness

1. Speech Rationale: Broca's area in the brain is responsible for the motor aspects of speech, through coordination of the muscular activity of the tongue, mouth, and larynx. The term assigned to damage in this area is aphasia. The items listed in the other options are not the responsibility of Broca's area.

The nurse is evaluating the respiratory outcomes for a client with Guillain-Barré syndrome. The nurse determines that which are acceptable outcomes for the client? Select all that apply. 1. Spontaneous breathing 2. Oxygen saturation of 98% 3. Adventitious breath sounds 4. Normal arterial blood gas levels 5. Vital capacity within normal range

1. Spontaneous breathing 2. Oxygen saturation of 98% 4. Normal arterial blood gas levels 5. Vital capacity within normal range Rationale: Satisfactory respiratory outcomes for a client with Guillain-Barré syndrome include clear breath sounds on auscultation, spontaneous breathing, normal vital capacity, normal arterial blood gas levels, and normal pulse oximetry. Adventitious breath sounds are an abnormal finding.

The nurse is caring for a client with respiratory failure related to Guillain-Barré syndrome. The nurse plans care knowing that what other extrapulmonary causes can lead to respiratory failure? Select all that apply. 1. Stroke 2. Pneumonia 3. Sleep apnea 4. Myasthenia gravis 5. Obstructive lung disease 6. Opioid analgesics, sedatives, anesthetics

1. Stroke 3. Sleep apnea 4. Myasthenia gravis 6. Opioid analgesics, sedatives, anesthetics Rationale: Extrapulmonary causes of respiratory failure include the following: stroke; sleep apnea; myasthenia gravis; and opioid analgesics, sedatives, and anesthetics. Both obstructive lung disease and pneumonia are intrapulmonary causes of respiratory failure.

The nurse is teaching a client with myasthenia gravis about the prevention of myasthenic and cholinergic crises. Which client activity suggests that teaching is most effective? 1. Taking medications as scheduled 2. Eating large, well-balanced meals 3. Doing muscle-strengthening exercises 4. Doing all chores early in the day while less fatigued

1. Taking medications as scheduled Rationale: Clients with myasthenia gravis are taught to space out activities over the day to conserve energy and restore muscle strength. Taking medications correctly to maintain blood levels that are not too low or too high is important. Muscle-strengthening exercises are not helpful and can fatigue the client. Overeating is a cause of exacerbation of symptoms, as is exposure to heat, crowds, erratic sleep habits, and emotional stress.

A client who had a stroke (brain attack) has right-sided hemianopsia. What should the nurse plan to do to help the client adapt to this problem? 1. Teach the client to scan the environment. 2. Place all objects within the left visual field. 3. Place all objects within the right visual field. 4. Ensure that the family brings the client's eyeglasses to hospital.

1. Teach the client to scan the environment. Rationale: Hemianopsia is blindness in half of the visual field. The client with hemianopsia is taught to scan the environment. This allows the client to take in the entirety of the visual field, which is necessary for proper functioning within the environment and helps to prevent injury to the client. Options 2 and 3 will not help the client adapt to this visual impairment. Eyeglasses are useful if the client already wears them, but they will not correct this visual field deficit.

The nurse witnesses an automobile crash on a highway and stops to provide assistance to the victim. The nurse notes that the client has sustained a head injury and a compound fracture to the left leg. The nurse provides the appropriate care before transport of the victim to the hospital by ambulance. The client develops a severe bone infection at the site of the fracture that requires amputation of the leg and files suit against the nurse who provided care at the scene of the crash. Which is accurate regarding the nurse's immunity from this suit? 1. The Good Samaritan law will protect the nurse. 2. The Good Samaritan law will not protect the nurse. 3. The Good Samaritan law protects laypersons but not professional primary health care providers (PHCPs). 4. The Good Samaritan law will provide immunity from the suit, even if the nurse has accepted compensation for the care provided.

1. The Good Samaritan law will protect the nurse. Rationale: A Good Samaritan law is passed by a state legislature to encourage nurses and other PHCPs to provide care to a person when an accident, emergency, or injury occurs without fear of being sued for the care provided. Its protection lies in the inability to sue the nurse or other PHCP for negligence in the care provided at the scene of the accident or during the emergency, even if further injury occurred because of the PHCP's care. Called immunity from suit, this protection usually applies only if all conditions of the law are met, such as that the PHCP received no compensation for the care provided and the care given was not willfully and wantonly negligent.

The nurse is assigned to care for a client with complete right-sided hemiparesis from a stroke (brain attack). Which characteristics are associated with this condition? Select all that apply. 1. The client is aphasic. 2. The client has weakness on the right side of the body. 3. The client has complete bilateral paralysis of the arms and legs. 4. The client has weakness on the right side of the face and tongue. 5. The client has lost the ability to move the right arm but is able to walk independently. 6. The client has lost the ability to ambulate independently but is able to feed and bathe herself or himself without assistance.

1. The client is aphasic. 2. The client has weakness on the right side of the body. 4. The client has weakness on the right side of the face and tongue. Rationale: Hemiparesis is a weakness of one side of the body that may occur after a stroke. It involves weakness of the face and tongue, arm, and leg on one side. These clients are also aphasic, unable to discriminate words and letters. They are generally very cautious and get anxious when attempting a new task. Complete bilateral paralysis does not occur in hemiparesis. The client with right-sided hemiparesis has weakness of the right arm and leg and needs assistance with feeding, bathing, and ambulating.

A client arrives in the hospital emergency department with a closed head injury to the right side of the head caused by an assault with a baseball bat. The nurse assesses the client neurologically, looking primarily for motor response deficits that involve which area? 1. The left side of the body 2. The right side of the body 3. Both sides of the body equally 4. Cranial nerves only, such as speech and pupillary response

1. The left side of the body Rationale: Motor responses such as weakness and decreased movement will be seen on the side of the body that is opposite an area of head injury. Contralateral deficits result from compression of the cortex of the brain or the pyramidal tracts. Depending on the severity of the injury, the client may have a variety of neurological deficits.

The nurse is creating a plan of care for a client with dysphagia following a stroke (brain attack). Which should the nurse include in the plan? Select all that apply. 1. Thicken liquids. 2. Assist the client with eating. 3. Assess for the presence of a swallow reflex. 4. Place the food on the affected side of the mouth. 5. Provide ample time for the client to chew and swallow.

1. Thicken liquids. 2. Assist the client with eating. 3. Assess for the presence of a swallow reflex. 5. Provide ample time for the client to chew and swallow. Rationale: Liquids are thickened to prevent aspiration. The nurse should assist the client with eating and place food on the unaffected side of the mouth. The nurse should assess for gag and swallowing reflexes before the client with dysphagia is started on a diet. The client should be allowed ample time to chew and swallow to prevent choking.

The nurse has completed client teaching on use of thrombolytic medications in acute ischemic stroke. The nurse determines that the educational session was effective if the client states that thrombolytics are used for what purpose? 1. To dissolve clots 2. To prevent ischemia 3. To prevent bleeding 4. To decrease anxiety

1. To dissolve clots Rationale: Thrombolytic medications are used to treat acute thrombolytic disorders. These medications dissolve clots. Because these medications alter the hemostatic capability of the client, any bleeding that does occur can be difficult to control. Options 2, 3, and 4 are not actions of this medication.

The nurse walking in a downtown business area witnesses a worker fall from a ladder. The nurse rushes to the victim who is unresponsive. A layperson is attempting to perform resuscitative measures. The nurse should intervene if which action by the layperson is noted? 1. Use of the head tilt-chin lift 2. Checking the scene for safety 3. Use of the jaw thrust maneuver 4. Moving the client away from a busy traffic road

1. Use of the head tilt-chin lift Rationale: Whenever a neck injury is suspected, the jaw thrust maneuver should be used during basic life support (BLS) to open the airway. The head tilt-chin lift produces hyperextension of the neck and could cause complications if a neck injury is present. The scene should be checked for safety, and the client should be moved away from a busy traffic road in order to ensure safety.

The client with a cervical spine injury has cervical tongs applied in the emergency department. What should the nurse include when planning care for this client? Select all that apply. 1. Using a RotoRest bed 2. Ensuring that weights hang freely 3. Removing the weights to reposition the client 4. Assessing the integrity of the weights and pulleys 5. Comparing the amount of prescribed traction with the amount in use

1. Using a RotoRest bed 2. Ensuring that weights hang freely 4. Assessing the integrity of the weights and pulleys 5. Comparing the amount of prescribed traction with the amount in use Rationale: Cervical tongs are applied after drilling holes in the client's skull under local anesthesia. Weights are attached to the tongs, which exert pulling pressure on the longitudinal axis of the cervical spine. Serial x-rays of the cervical spine are taken, with weights being added gradually until the x-ray reveals that the vertebral column is realigned. After that, weights may be reduced gradually to a point that maintains alignment. The client with cervical tongs is placed on a Stryker frame or RotoRest bed. The nurse ensures that weights hang freely and the amount of weight matches the current prescription. The nurse also inspects the integrity and position of the ropes and pulleys. The nurse does not remove the weights to administer care.

A client has a cerebellar lesion. The nurse should plan to obtain which item for use by the client? 1. Walker 2. Slider board 3. Raised toilet seat 4. Adaptive eating utensils

1. Walker Rationale: The cerebellum is responsible for balance and coordination. A walker would provide stability for the client during ambulation. A slider board would be used in transferring a client with weak or paralyzed legs from a bed to a stretcher or wheelchair. A raised toilet seat would be useful if the client did not have sufficient mobility or ability to flex the hips. Adaptive eating utensils would be beneficial if the client had partial paralysis of the hand.

The nurse is providing medication instructions to a client with multiple sclerosis receiving baclofen. Which information should the nurse include in the instructions? 1. Watch for urinary retention as a side effect. 2. Stop taking the medication if diarrhea occurs. 3. Restrict fluid intake while taking this medication. 4. Notify the primary health care provider if fatigue occurs.

1. Watch for urinary retention as a side effect. Rationale: Baclofen, a skeletal muscle relaxant, also is a central nervous system (CNS) depressant, which can cause urinary retention. The client should not restrict fluid intake. Constipation, rather than diarrhea, is an adverse effect of baclofen. Fatigue is a CNS effect that is most intense during the early phase of therapy and diminishes with continued medication use. It is not necessary to notify the primary health care provider if fatigue occurs.

The nurse is assessing the function of cranial nerve XII in a client who sustained a stroke. To assess function of this nerve, which action should the nurse ask the client to perform? 1. Extend the arms. 2. Extend the tongue. 3. Turn the head toward the nurse's arm. 4. Focus the eyes on the object held by the nurse.

2. Extend the tongue. Rationale: Impairment of cranial nerve XII can occur with a stroke. To assess the function of cranial nerve XII (the hypoglossal nerve), the nurse would assess the client's ability to extend the tongue. The maneuvers noted in the remaining options do not test the function of cranial nerve XII

The nurse has given medication instructions to a client receiving phenytoin. Which statement indicates that the client has an adequate understanding of the instructions? 1. "Alcohol is not contraindicated while taking this medication." 2. "Good oral hygiene is needed, including brushing and flossing." 3. "The medication dose may be self-adjusted, depending on side effects." 4. "The morning dose of the medication should be taken before a serum medication level is drawn."

2. "Good oral hygiene is needed, including brushing and flossing." Rationale: Typical antiseizure medication instructions include taking the prescribed daily dosage to keep the blood level of the medication constant and having a sample drawn for serum medication level determination before taking the morning dose. The client is taught not to stop the medication abruptly, to avoid alcohol, to check with a primary health care provider before taking over-the-counter medications, to avoid activities in which alertness and coordination are required until medication effects are known, to provide good oral hygiene, and to obtain regular dental care. The client should also wear a MedicAlert bracelet.

The nurse has completed discharge instructions for a client with application of a halo device who sustained a cervical spinal cord injury. Which statement indicates that the client needs further clarification of the instructions? 1. "I will use a straw for drinking." 2. "I will drive only during the daytime." 3. "I will be careful because the device alters balance." 4. "I will wash the skin daily under the lamb's wool liner of the vest."

2. "I will drive only during the daytime." Rationale: The halo device alters balance and can cause fatigue because of its weight. The client should cleanse the skin daily under the vest to protect the skin from ulceration and should avoid the use of powder or lotions. The liner should be changed if odor becomes a problem. The client should have food cut into small pieces to facilitate chewing and use a straw for drinking. Pin care is done as instructed. The client cannot drive at all, because the device impairs the range of vision.

The nurse has given the client with Bell's palsy instructions on preserving muscle tone in the face and preventing denervation. The nurse determines that the client needs further teaching if the client makes which statements? 1. "I will perform facial exercises." 2. "I will expose my face to cold to decrease the pain." 3. "I will massage my face with a gentle upward motion." 4. "I will wrinkle my forehead, blow out my cheeks, and whistle frequently."

2. "I will expose my face to cold to decrease the pain." Rationale: Exposure to cold or drafts is avoided in Bell's palsy because it can cause discomfort. Prevention of muscle atrophy with Bell's palsy is accomplished with facial massage, facial exercises, and electrical nerve stimulation. Local application of heat to the face may improve blood flow and provide comfort.

The nurse is admitting a client to the hospital emergency department from a nursing home. The client is unconscious with an apparent frontal head injury. A medical diagnosis of epidural hematoma is suspected. Which question is of the highest priority for the emergency department nurse to ask of the transferring nurse at the nursing home? 1. "When did the injury occur?" 2. "Was the client awake and talking right after the injury?" 3. "What medications has the client received since the fall?" 4. "What was the client's level of consciousness before the injury?"

2. "Was the client awake and talking right after the injury?" Rationale: Epidural hematomas frequently are characterized by a "lucid interval" that lasts for minutes to hours, during which the client is awake and talking. After this lucid interval, signs and symptoms progress rapidly, with potentially catastrophic intracranial pressure increase. Epidural hematomas are medical emergencies. It is important for the nurse to assist in the differentiation between epidural hematoma and other types of head injuries.

The nurse is observing a new nursing graduate who is preparing an intermittent intravenous (IV) infusion of phenytoin for a client with a diagnosis of seizures. Which solution used by the nursing graduate should indicate to the nurse an understanding of proper preparation of this medication? 1. 5% dextrose in water 2. 0.9% sodium chloride 3. Lactated Ringer's solution 4. 5% dextrose and 0.45% sodium chloride

2. 0.9% sodium chloride Rationale: Intermittent IV infusion of phenytoin is administered by injection into a large vein, using normal saline solution. Dextrose solutions are avoided because the medication will precipitate in these solutions. Therefore, the options containing dextrose identify incorrect solutions for IV administration with this medication. In addition, lactated Ringer's solution contains electrolytes that can interfere with phenytoin administration.

The home care nurse is making extended follow-up visits to a client discharged from the hospital after a moderately severe head injury. The family states that the client is behaving differently than before the accident. The client is more fatigued and irritable and has some memory problems. The client, who was previously very even tempered, is prone to outbursts of temper now. The nurse determines that these behaviors are indicative of which problem? 1. Intracranial pressure changes 2. A long-term sequela of the injury 3. A worsening of the original injury 4. A short-term problem that will resolve in about 1 month

2. A long-term sequela of the injury Rationale: Clients with moderate to severe head injury usually have residual physical and cognitive disabilities; these include personality changes, increased fatigue and irritability, mood alterations, and memory changes. The client also may require frequent to constant supervision. The nurse assesses the family's ability to cope and makes appropriate referrals to respite services, support groups, and state or local chapters of the National Head Injury Foundation.

A client with myasthenia gravis is having difficulty with airway clearance and difficulty with maintaining an effective breathing pattern. The nurse should keep which most important items available at the client's bedside? 1. Oxygen and metered-dose inhaler 2. Ambu bag and suction equipment 3. Pulse oximeter and cardiac monitor 4. Incentive spirometer and cough pillow

2. Ambu bag and suction equipment Rationale: The client with myasthenia gravis may experience episodes of respiratory distress if excessively fatigued or with development of myasthenic or cholinergic crisis. For this reason, an Ambu bag, intubation tray, and suction equipment should be available at the bedside.

The primary health care provider is preparing to administer edrophonium to the client with myasthenia gravis. In planning care, the nurse understands which about the administration of edrophonium? Select all that apply. 1. Edrophonium is a long-acting cholinesterase inhibitor. 2. Atropine is used to reverse the effects of edrophonium. 3. If symptoms worsen following administration of edrophonium, the crisis is cholinergic. 4. Edrophonium is used to distinguish between a myasthenic crisis and a cholinergic crisis. 5. An improvement in symptoms following administration of edrophonium indicates myasthenic crisis.

2. Atropine is used to reverse the effects of edrophonium. 3. If symptoms worsen following administration of edrophonium, the crisis is cholinergic. 4. Edrophonium is used to distinguish between a myasthenic crisis and a cholinergic crisis. 5. An improvement in symptoms following administration of edrophonium indicates myasthenic crisis. Rationale: Edrophonium is an ultra-short-acting reversible cholinesterase inhibitor that can be used to distinguish between a cholinergic and a myasthenic crisis. To distinguish between overtreatment (cholinergic crisis) and undertreatment (myasthenic crisis), edrophonium is administered; this is often referred to as a Tensilon test. Overtreatment of myasthenia gravis with reversible cholinesterase inhibitors results in a cholinergic crisis. Undertreatment can result in a myasthenic crisis. Both cholinergic and myasthenic crises result in increased muscle weakness or paralysis. If symptoms improve after the administration of edrophonium, the crisis is myasthenic; if symptoms worsen, the crisis is cholinergic. Atropine must be readily available so that edrophonium can be reversed if the symptoms worsen.

A child is admitted with a head injury after being in a motor vehicle accident. After noting the presence of clear drainage from the left ear, the nurse would suspect which underlying problem commonly associated with this finding? 1. Linear skull fracture 2. Basilar skull fracture 3. Subdural hematoma 4. Epidural hematoma

2. Basilar skull fracture Rationale: Drainage of cerebrospinal fluid (clear fluid) from the ear is a symptom of basilar skull fracture. Linear skull fractures are often asymptomatic. Subdural and epidural hematomas often present with s/s of increasing ICP.

The nurse is administering medications to a client with trigeminal neuralgia. The nurse expects that which medication will be prescribed for pain relief? 1. Oxycodone plus aspirin 2. Carbamazepine and gabapentin 3. Acetaminophen and codeine sulfate 4. Meperidine hydrochloride and hydroxyzine

2. Carbamazepine and gabapentin Rationale: The anticonvulsant medications carbamazepine and gabapentin help relieve the pain in many clients with trigeminal neuralgia. They act by inhibiting the reactivity of neurons in the trigeminal nerve. Opioid analgesics (oxycodone, codeine sulfate, and meperidine hydrochloride) are not very effective in controlling pain caused by trigeminal neuralgia.

A client who has been taking phenytoin for seizure control has a serum phenytoin level of 8 mcg/mL (35.71 mmol/L). On the basis of this finding, which note should the nurse enter in the client's health record? 1. Client is experiencing a toxic level. 2. Client has an inadequate medication level. 3. Client's result is at the low end of therapeutic range. 4. Client's result is at the high end of therapeutic range.

2. Client has an inadequate medication level. Rationale: The therapeutic serum level range for phenytoin is 10 to 20 mcg/mL (40 to 79 mmol/L). A laboratory value of 8 mcg/mL is below the therapeutic range, indicating an inadequate medication level, so this should be noted in the health record and the primary health care provider should be notified.

A client who suffered a stroke is prepared for discharge from the hospital. The primary health care provider has prescribed range-of-motion (ROM) exercises for the client's right side. What action should the nurse include in the client's plan of care? 1. Implement ROM exercises to the point of pain for the client. 2. Consider the use of active, passive, or active-assisted exercises in the home. 3. Encourage the client to be dependent on the home care nurse to complete the exercise program. 4. Develop a schedule of ROM exercises every 2 hours while awake even if the client is fatigued.

2. Consider the use of active, passive, or active-assisted exercises in the home. Rationale: The home care nurse must consider all forms of ROM for the client. Even a client with hemiplegia can participate in some components of rehabilitative care. In addition, the goal in home care nursing is for the client to assume as much self-care and independence as possible. The nurse needs to teach home care measures so that the client becomes self-reliant. The options of performing ROM exercises to the point of pain and performing ROM exercises every 2 hours while the client is awake even if fatigued are incorrect from a physiological standpoint.

The nurse assesses a client who is diagnosed with a stroke (brain attack). On assessment, the client is unable to understand the nurse's commands. Which condition should the nurse document? 1. Occipital lobe impairment 2. Damage to the auditory association areas 3. Frontal lobe and optic nerve tracts damage 4. Difficulty with concept formation and abstraction areas

2. Damage to the auditory association areas Rationale: Auditory association and storage areas are located in the temporal lobe and relate to understanding spoken language. The occipital lobe contains areas related to vision. The frontal lobe controls voluntary muscle activity, including speech, and an impairment can result in expressive aphasia. The parietal lobe contains association areas for concept formation, abstraction, spatial orientation, body and object size and shape, and tactile sensation.

The nurse is performing an assessment on a client with a head injury and notes that the client is assuming this posture. The nurse contacts the primary health care provider and reports that the client is exhibiting which posture? Refer to figure. 1. Opisthotonos 2. Decorticate rigidity 3. Decerebrate rigidity 4. Flaccid quadriplegia

2. Decorticate rigidity Rationale: In decorticate rigidity, the upper extremities (arms, wrists, and fingers) are flexed with adduction of the arms. The lower extremities are extended with internal rotation and plantar flexion. Decorticate rigidity indicates a hemispheric lesion of the cerebral cortex. Opisthotonos is prolonged arching of the back with the head and heels bent backward. Opisthotonos indicates meningeal irritation. In decerebrate rigidity, the upper extremities are stiffly extended and adducted with internal rotation and pronation of the palms. The lower extremities are stiffly extended with plantar flexion. The teeth are clenched, and the back is hyperextended. Decerebrate rigidity indicates a lesion in the brainstem at the midbrain or upper pons. Flaccid quadriplegia is complete loss of muscle tone and paralysis of all four extremities, indicating a completely nonfunctional brainstem.

A client with multiple sclerosis is receiving baclofen. The nurse assessing the client monitors for which finding as an indication of a primary therapeutic response to the medication? 1. Decreased nausea 2. Decreased muscle spasms 3. Increased muscle tone and strength 4. Increased range of motion of all extremities

2. Decreased muscle spasms Rationale: Baclofen is a skeletal muscle relaxant and acts at the spinal cord level to decrease the frequency and amplitude of muscle spasms in clients with spinal cord injuries or diseases or with multiple sclerosis. Increased muscle tone and strength and increased range of motion of all extremities are not directly related to the effects of this medication. Decreased nausea is an incorrect option.

The client with a traumatic brain injury (TBI) has begun excreting copious amounts of dilute urine through the Foley catheter. The client's urine output for the previous shift was 3000 mL. The nurse expects that the primary health care provider will prescribe which medication? 1. Mannitol 2. Desmopressin 3. Ethacrynic acid 4. Dexamethasone

2. Desmopressin Rationale: A complication of TBI is diabetes insipidus, which can occur with insult to the hypothalamus, the antidiuretic hormone storage vesicles, or the posterior pituitary gland. Urine output that exceeds 9 L per day generally requires treatment with desmopressin. Dexamethasone is usually given to control cerebral edema secondary to brain tumors. Ethacrynic acid and mannitol are diuretics, which would be contraindicated.

A client is diagnosed with Bell's palsy. The nurse assessing the client expects to note which symptom? 1. A symmetrical smile 2. Difficulty closing the eyelid on the affected side 3. Narrowing of the palpebral fissure on the affected side 4. Paroxysms of excruciating pain in the lips and cheek on the affected side

2. Difficulty closing the eyelid on the affected side Rationale: The facial drooping associated with Bell's palsy makes it difficult for the client to close the eyelid on the affected side. A widening of the palpebral fissure (the opening between the eyelids) and an asymmetrical smile are seen with Bell's palsy. Paroxysms of excruciating pain are characteristic of trigeminal neuralgia

A client with multiple sclerosis is receiving diazepam, a centrally acting skeletal muscle relaxant. Which finding, if noted during the nursing assessment, would indicate that the client is experiencing a side/adverse effect of this medication? 1. Headache 2. Drowsiness 3. Urinary retention 4. Increased salivation

2. Drowsiness Rationale: Incoordination and drowsiness are common side/adverse effects of diazepam. The remaining options are unrelated to the use of this medication.

Which assessment finding should the nurse expect to note in the client hospitalized with a diagnosis of stroke who has difficulty chewing food? 1. Dysfunction of vagus nerve (cranial nerve X) 2. Dysfunction of trigeminal nerve (cranial nerve V) 3. Dysfunction of hypoglossal nerve (cranial nerve XII) 4. Dysfunction of spinal accessory nerve (cranial nerve XI)

2. Dysfunction of trigeminal nerve (cranial nerve V) Rationale: The motor branch of cranial nerve V is responsible for the ability to chew food. The vagus nerve is active in parasympathetic functions of the autonomic nervous system. The hypoglossal nerve aids in swallowing. The spinal accessory nerve is responsible for shoulder movement, among other things.

A client is newly admitted to the hospital with a diagnosis of stroke (brain attack) manifested by complete hemiplegia. Which item in the medical history of the client should the nurse be most concerned about? 1. Glaucoma 2. Emphysema 3. Hypertension 4. Diabetes mellitus

2. Emphysema Rationale: The nurse should be most concerned about emphysema. The respiratory system is the priority in the acute phase of a stroke. The client with a stroke is vulnerable to respiratory complications such as atelectasis and pneumonia. Because the client has complete hemiplegia (is unable to move) and has emphysema, these risks are very significant. Although the other conditions of glaucoma, hypertension, and diabetes mellitus are important, they are not as significant as emphysema.

The nurse is providing discharge education to a client diagnosed with trigeminal neuralgia. Which medication will likely be prescribed upon discharge for this condition? 1. Lorazepam 2. Gabapentin 3. Carisoprodol 4. Chlordiazepoxide

2. Gabapentin Rationale: Trigeminal neuralgia is characterized by spasms of pain that start suddenly and last from seconds to minutes. The pain often is described as either stabbing or similar to an electric shock. It is accompanied by spasms of the facial muscles that cause twitching of parts of the face or mouth, or closure of the eye. It is treated by giving antiseizure medications, such as gabapentin, and sometimes tricyclic antidepressants. These medications work by stabilizing the neuronal membrane and blocking the nerve.

A client has a closed head injury with increased intracranial pressure (ICP). The increased ICP is being managed by mannitol 25 g by the intravenous (IV) route every 2 hours. The nurse is planning to administer this medication via IV pump in what manner? 1. Mixed in solution with the IV antibiotics 2. Giving it slowly over 30 to 90 minutes 3. Piggybacked into the packed red blood cells 4. Giving it rapidly over 5 minutes by IV bolus

2. Giving it slowly over 30 to 90 minutes Rationale: Mannitol is an osmotic diuretic. When used to treat increased ICP, it is given slowly over 30 to 90 minutes, not rapidly and not via IV bolus. Mannitol should not be mixed in solutions with antibiotics, and nothing should be piggybacked with packed red blood cells.

The nurse is instructing a client who had a stroke and has weakness on 1 side how to ambulate with the use of a cane. Which instruction should the nurse provide to the client? 1. Hold the cane on the affected (weak) side. 2. Hold the cane on the unaffected (strong) side. 3. Move the cane forward first along with the unaffected (strong) leg. 4. Move the cane and the unaffected (strong) leg down first when going down stairs.

2. Hold the cane on the unaffected (strong) side. Rationale: The cane is kept on the strong side of the body. It would be hard to hold the cane on the weak side. The cane is assisting the weakened leg, so the weakened leg moves with the cane, or right after it, in ambulating or in going down stairs.

A client who has a closed-head injury has ICP readings that ranged from 16 to 22 mmHg for the past 2 days. Which of the following actions should the nurse take to decrease the potential for raising the client's ICP? Select all that apply. 1. Suction endotracheal tube using a closed system. 2. Hyperventilate the client. 3. Elevate the client's head using two pillows. 4. Administer stool softener. 5. Keep the client well hydrated.

2. Hyperventilate the client. 4. Administer stool softener. Rationale: Hyperventilation of the client will prevent hypercarbia which can cause vasodilation with a secondary increase in ICP. Admin of stool softener will also decrease the need to bear down during bowel movements. Hyperflexion of the client's neck with pillows and over hydration all carry the risk of increasing ICP and should be avoided. Suctioning also increases ICP and should be done only when indicated.

A client who has increased ICP has been prescribed mannitol (Osmitrol) IV. For which of the following side effects should the nurse monitor? 1. Hyperglycemia 2. Hyponatremia 3. Hypervolemia 4. Oliguria

2. Hyponatremia Rationale: Mannitol is a powerful osmotic diuretic that carries the risk of fluid and electrolyte imbalances such as hyponatremia. Hyperglycemia is not a side effect of mannitol. Hypovolemia and polyuria are side effects as opposed to hypervolemia and oliguria.

The nurse notes that a client who has suffered a brain injury has an adequate heart rate, blood pressure, fluid balance, and body temperature. Based on these clinical findings, the nurse determines that which brain area is functioning properly? 1. Thalamus 2. Hypothalamus 3. Limbic system 4. Reticular activating system

2. Hypothalamus Rationale: The hypothalamus is responsible for autonomic nervous system functions, such as heart rate, blood pressure, temperature, and fluid and electrolyte balance (among others). The thalamus acts as a relay station for sensory and motor information. The limbic system is responsible for emotions. The reticular activating system is responsible for the sleep-wake cycle.

The nurse is caring for the client with increased intracranial pressure as a result of a head injury? The nurse would note which trend in vital signs if the intracranial pressure is rising? 1. Increasing temperature, increasing pulse, increasing respirations, decreasing blood pressure 2. Increasing temperature, decreasing pulse, decreasing respirations, increasing blood pressure 3. Decreasing temperature, decreasing pulse, increasing respirations, decreasing blood pressure 4. Decreasing temperature, increasing pulse, decreasing respirations, increasing blood pressure

2. Increasing temperature, decreasing pulse, decreasing respirations, increasing blood pressure Rationale: A change in vital signs may be a late sign of increased intracranial pressure. Trends include increasing temperature and blood pressure and decreasing pulse and respirations. Respiratory irregularities also may occur.

A client is admitted with an exacerbation of multiple sclerosis. The nurse is assessing the client for possible precipitating risk factors. Which factor, if reported by the client, should the nurse identify as being unrelated to the exacerbation? 1. Annual influenza vaccination 2. Ingestion of increased fruits and vegetables 3. An established routine of walking 2 miles each evening 4. A recent period of extreme outside ambient temperatures

2. Ingestion of increased fruits and vegetables Rationale: The onset or exacerbation of multiple sclerosis can be preceded by a number of different factors, including physical stress (e.g., vaccination, excessive exercise), emotional stress, fatigue, infection, physical injury, pregnancy, extremes in environmental temperature, and high humidity. No methods of primary prevention are known. Intake of fruits and vegetables is a healthy and an unrelated item.

The nurse is performing an assessment on a client with the diagnosis of Brown-Séquard syndrome. The nurse would expect to note which assessment finding? 1. Bilateral loss of pain and temperature sensation 2. Ipsilateral paralysis and loss of touch and vibration 3. Contralateral paralysis and loss of touch, pressure, and vibration 4. Complete paraplegia or quadriplegia, depending on the level of injury

2. Ipsilateral paralysis and loss of touch and vibration Rationale: Brown-Séquard syndrome results from hemisection of the spinal cord, resulting in ipsilateral paralysis and loss of touch, pressure, vibration, and proprioception. Contralaterally, pain and temperature sensation are lost because these fibers decussate after entering the cord. The remaining options are not assessment findings in this syndrome.

The home care nurse is making a visit to a client who requires use of a wheelchair after a spinal cord injury sustained 4 months earlier. Just before leaving the home, the nurse ensures that which intervention has been done to prevent an episode of autonomic dysreflexia (hyperreflexia)? 1. Updating the home safety sheet 2. Leaving the client in an unchilled area of the room 3. Noting a bowel movement on the client progress note 4. Recording the amount of urine obtained with catheterization

2. Leaving the client in an unchilled area of the room Rationale: The most common cause of autonomic dysreflexia is visceral stimuli, such as with blockage of urinary drainage or with constipation. Barring these, other causes include noxious mechanical and thermal stimuli, particularly pressure and overchilling. For this reason, the nurse ensures that the client is positioned with no pinching or pressure on paralyzed body parts and that the client will be sufficiently warm.

A client with a spinal cord injury is prone to experiencing autonomic dysreflexia. The nurse would avoid which of the following measures to minimize the risk of recurrence? 1. Strict adherence to a bowel retraining program 2. Limiting bladder catheterization to once every 12 hours 3. Keeping the linen wrinkle-free 4. Preventing unnecessary pressure on the lower limbs

2. Limiting bladder catheterization to once every 12 hours Rationale: Need to be cathing more frequently than every 12 hours. Should be every 4-6 hours.

A client admitted to the nursing unit from the hospital emergency department has a C4 spinal cord injury. In conducting the admission assessment, what is the nurse's priority action? 1. Take the temperature. 2. Listen to breath sounds. 3. Observe for dyskinesias. 4. Assess extremity muscle strength.

2. Listen to breath sounds. Rationale: Because compromise of respiration is a leading cause of death in cervical cord injury, respiratory assessment is the highest priority. Assessment of temperature and strength can be done after adequate oxygenation is ensured. Because dyskinesias occur in cerebellar disorders, this is not as important a concern as in cord-injured clients unless head injury is suspected.

The nurse in the health care clinic is providing medication instructions to a client with a seizure disorder who will be taking divalproex sodium. The nurse should instruct the client about the importance of returning to the clinic for monitoring of which laboratory study? 1. Electrolyte panel 2. Liver function studies 3. Renal function studies 4. Blood glucose level determination

2. Liver function studies Rationale: Divalproex sodium, an anticonvulsant, can cause fatal hepatotoxicity. The nurse should instruct the client about the importance of monitoring the results of liver function studies and ammonia level determinations. The studies in the remaining options are not required with the use of this medication.

The nurse is performing an assessment on a client with Guillain-Barré syndrome. The nurse determines that which finding would be of most concern? 1. Difficulty articulating words 2. Lung vital capacity of 10 mL/kg 3. Paralysis progressing from the toes to the waist 4. A blood pressure (BP) decrease from 110/78 mm Hg to 102/70 mm Hg

2. Lung vital capacity of 10 mL/kg Rationale: Respiratory compromise is a major concern in clients with Guillain-Barré syndrome. Clients often are intubated and mechanically ventilated when the vital capacity is less than 15 mL/kg. Difficulty articulating words and paralysis progressing from the toes to the waist are expected, depending on the degree of paralysis that occurs. Although orthostatic hypotension is a problem with these clients, the BP drop in option 4 is less than 10 mm Hg and is not significant.

The nurse is reviewing the record for a client seen in the health care clinic and notes that the primary health care provider has documented a diagnosis of amyotrophic lateral sclerosis (ALS). Which initial clinical manifestation of this disorder should the nurse expect to see documented in the record? 1. Muscle wasting 2. Mild clumsiness 3. Altered mentation 4. Diminished gag reflex

2. Mild clumsiness Rationale: The initial symptom of ALS is a mild clumsiness, usually noted in the distal portion of 1 extremity. The client may complain of tripping and drag 1 leg when the lower extremities are involved. Mentation and intellectual function usually are normal. Diminished gag reflex and muscle wasting are not initial clinical manifestations.

A client was seen and treated in the hospital emergency department for a concussion. The nurse determines that family members need further teaching if they verbalize to call the primary health care provider (PHCP) for which client sign or symptom? 1. Vomiting 2. Minor headache 3. Difficulty speaking 4. Difficulty awakening

2. Minor headache Rationale: A concussion after head injury is a temporary loss of consciousness (from a few seconds to a few minutes) without evidence of structural damage. After concussion, the family is taught to monitor the client and call the PHCP or return the client to the emergency department for signs and symptoms such as confusion, difficulty awakening or speaking, one-sided weakness, vomiting, and severe headache. Minor headache is expected.

The nurse is assessing the motor and sensory function of an unconscious client who sustained a head injury. The nurse should use which technique to test the client's peripheral response to pain? 1. Sternal rub 2. Nailbed pressure 3. Pressure on the orbital rim 4. Squeezing of the sternocleidomastoid muscle

2. Nailbed pressure Rationale: Nailbed pressure tests a basic motor and sensory peripheral response. Cerebral responses to pain are tested using a sternal rub, placing upward pressure on the orbital rim, or squeezing the clavicle or sternocleidomastoid muscle.

A client with a traumatic brain injury is able, with eyes closed, to identify a set of keys placed in his or her hands. On the basis of this assessment finding, the nurse determines that there is appropriate function of which lobe of the brain? 1. Frontal 2. Parietal 3. Occipital 4. Temporal

2. Parietal Rationale: The ability to distinguish an object by touch is called stereognosis, which is a function of the right parietal area. The parietal lobe of the brain is responsible for spatial orientation and awareness of sizes and shapes. The left parietal area is responsible for mathematics and right-left orientation. The other lobes of the brain are not responsible for this function.

The nurse develops a plan of care for a client with a brain aneurysm who will be placed on aneurysm precautions. Which interventions should be included in the plan? Select all that apply. 1. Leave the lights on in the client's room at night. 2. Place a blood pressure cuff at the client's bedside. 3. Close the shades in the client's room during the day. 4. Allow the client to drink 1 cup of caffeinated coffee a day. 5. Allow the client to ambulate 4 times a day with assistance.

2. Place a blood pressure cuff at the client's bedside. 3. Close the shades in the client's room during the day. Rationale: Aneurysm precautions include placing the client on bed rest in a quiet setting. The use of lights is kept to a minimum to prevent environmental stimulation. The nurse should monitor the blood pressure and note any changes that could indicate rupture. Any activity, such as pushing, pulling, sneezing, or straining, that increases the blood pressure or impedes venous return from the brain is prohibited. The nurse provides physical care to minimize increases in blood pressure. Visitors, radio, television, and reading materials are restricted or limited. Stimulants, such as nicotine and coffee and other caffeine-containing products, are prohibited. Decaffeinated coffee or tea may be used.

The nurse has a prescription to give dexamethasone by the intravenous (IV) route to a client with cerebral edema. How should the nurse prepare this medication? 1. Diluting the medication in 500 mL of 5% dextrose 2. Preparing an undiluted direct injection of the medication 3. Diluting the medication in 1 mL of lactated Ringer's solution for direct injection 4. Diluting the medication in 10% dextrose in water and administering it as a direct injection

2. Preparing an undiluted direct injection of the medication Rationale: Dexamethasone may be given by direct IV injection or IV infusion. For IV infusion, it may be mixed with 50 to 100 mL of 0.9% sodium chloride or 5% dextrose in water. It is not mixed with lactated Ringer's solution or 10% dextrose in water.

A client has sustained damage to Wernicke's area from a stroke (brain attack). On assessment of the client, which sign or symptom would be noted? 1. Difficulty speaking 2. Problem with understanding language 3. Difficulty controlling voluntary motor activity 4. Problem with articulating events from the remote past

2. Problem with understanding language Rationale: Wernicke's area consists of a small group of cells in the temporal lobe whose function is the understanding of language. Damage to Broca's area is responsible for aphasia. The motor cortex in the precentral gyrus controls voluntary motor activity. The hippocampus is responsible for the storage of memory.

At the end of the work shift, the nurse is reviewing the respiratory status of a client admitted with a stroke (brain attack) earlier in the day. The nurse determines that the client's airway is patent if which data are identified? 1. Respiratory rate 24 breaths/min, oxygen saturation 94%, breath sounds clear 2. Respiratory rate 18 breaths/min, oxygen saturation 98%, breath sounds clear 3. Respiratory rate 16 breaths/min, oxygen saturation 85%, wheezes bilaterally 4. Respiratory rate 20 breaths/min, oxygen saturation 92%, diminished breath sounds in lung bases

2. Respiratory rate 18 breaths/min, oxygen saturation 98%, breath sounds clear Rationale: The client's airway is most protected if all of the respiratory parameters measured fall within normal limits. Therefore, the respiratory rate should ideally be 16 to 20 breaths/min, the oxygen saturation should be greater than 95%, and the breath sounds should be clear. The correct option is the only one that meets all 3 criteria

The nurse is documenting nursing observations in the record of a client who experienced a tonic-clonic seizure. Which clinical manifestation did the nurse most likely note in the clonic phase of the seizure? 1. Body stiffening 2. Spasms of the entire body 3. Sudden loss of consciousness 4. Brief flexion of the extremities

2. Spasms of the entire body Rationale: The clonic phase of a seizure is characterized by alternating spasms and momentary muscular relaxation of the entire body, accompanied by strenuous hyperventilation. The face is contorted and the eyes roll. Excessive salivation results in frothing from the mouth. The tongue may be bitten, the client sweats profusely, and the pulse is rapid. The clonic jerking subsides by slowing in frequency and losing strength of contractions over a period of 30 seconds. Body stiffening, sudden loss of consciousness, and brief flexion of the extremities are associated with the tonic phase of a seizure.

The nurse is caring for a client who was admitted for a stroke (brain attack) of the temporal lobe. Which clinical manifestations should the nurse expect to note in the client? 1. The client will be unable to recall past events. 2. The client will have difficulty understanding language. 3. The client will have difficulty moving 1 side of the body. 4. The client will demonstrate difficulty articulating words.

2. The client will have difficulty understanding language. Rationale: Wernicke's area consists of a small group of cells in the temporal lobe, the function of which is the understanding of language. The hippocampus is responsible for the storage of memory (the client will be unable to recall past events). Damage to Broca's area is responsible for aphasia (the client will demonstrate difficulty articulating words). The motor cortex in the precentral gyrus controls voluntary motor activity (the client will have difficulty moving one side of the body).

A client who has had a stroke (brain attack) has residual dysphagia. When a diet prescription is initiated, the nurse should take which actions? Select all that apply. 1. Giving the client thin liquids 2. Thickening liquids to the consistency of oatmeal 3. Placing food on the unaffected side of the mouth 4. Allowing plenty of time for chewing and swallowing 5. Leave the client alone so that the client will gain independence by feeding self

2. Thickening liquids to the consistency of oatmeal 3. Placing food on the unaffected side of the mouth 4. Allowing plenty of time for chewing and swallowing Rationale: The client with dysphagia is started on a diet only after the gag and swallow reflexes have returned. The client is assisted with meals as needed and is given ample time to chew and swallow. Food is placed on the unaffected side of the mouth. Liquids are thickened to avoid aspiration. The client is not left alone because of the risk of aspiration.

The nurse is monitoring ongoing care for a potential organ donor who has been diagnosed with brain death. Which finding indicates to the nurse that the standard for ongoing care has been maintained? 1. PaO2 70 mm Hg 2. Urine output 100 mL/hr 3. Heart rate 52 beats/min 4. Blood pressure 90/48 mm Hg

2. Urine output 100 mL/hr Rationale: Adequate perfusion must be maintained to all vital organs in order for the client to remain viable as an organ donor. Guidelines may be used to maintain organ viability, but adequate perfusion is necessary. The correct option is the only one that indicates adequate perfusion. The incorrect options identify lower than normal values, thus adequate perfusion would not be maintained.

The nurse is planning discharge teaching for a client started on acetazolamide for a supratentorial lesion. Which information about the primary action of the medication should be included in the client's education? 1. It will prevent hypertension. 2. It will prevent hyperthermia. 3. It decreases cerebrospinal fluid production. 4. It maintains adequate blood pressure for cerebral perfusion.

3. It decreases cerebrospinal fluid production. Rationale: Acetazolamide is a carbonic anhydrase inhibitor and a diuretic. It is used in the client with or at risk for increased intracranial pressure to decrease cerebrospinal fluid production. The remaining options are not actions of this medication.

The home health nurse is visiting a client with a diagnosis of multiple sclerosis. The client has been taking oxybutynin. The nurse evaluates the effectiveness of the medication by asking the client which assessment question? 1. "Are you consistently fatigued?" 2. "Are you having muscle spasms?" 3. "Are you getting up at night to urinate?" 4. "Are you having normal bowel movements?"

3. "Are you getting up at night to urinate?" Rationale: Oxybutynin is an antispasmodic used to relieve symptoms of urinary urgency, frequency, nocturia, and incontinence in clients with uninhibited or reflex neurogenic bladder. Expected effects include improved urinary control and decreased urinary frequency, incontinence, and nocturia. The questions in the remaining options are unrelated to the use of this medication.

The home care nurse is performing an assessment on a client with a diagnosis of Bell's palsy. Which assessment question will elicit specific information regarding this client's disorder? 1. "Do your eyes feel dry?" 2. "Do you have any spasms in your throat?" 3. "Are you having any difficulty chewing food?" 4. "Do you have any tingling sensations around your mouth?"

3. "Are you having any difficulty chewing food?" Rationale: Bell's palsy is a one-sided facial paralysis caused by compression of the facial nerve. Manifestations include facial droop from paralysis of the facial muscles; increased lacrimation; painful sensations in the eye, face, or behind the ear; and speech or chewing difficulties.

The nurse has instructed a client with myasthenia gravis about strategies for self-management at home. The nurse determines a need for further teaching if the client makes which statement? 1. "Here's the MedicAlert bracelet I obtained." 2. "I should take my medications an hour before mealtime." 3. "Going to the beach will be a nice, relaxing form of activity." 4. "I've made arrangements to get a portable resuscitation bag and home suction equipment."

3. "Going to the beach will be a nice, relaxing form of activity." Rationale: Most ongoing treatment for myasthenia gravis is done in outpatient settings, and the client must be aware of the lifestyle changes needed to maintain independence. The client should carry medical identification about the presence of the condition. Taking medications an hour before mealtime gives greater muscle strength for chewing and is indicated. The client should have portable suction equipment and a portable resuscitation bag available in case of respiratory distress. The client should avoid situations and other factors, including stress, infection, heat, surgery, and alcohol, that could worsen the symptoms

The nurse has provided instructions to a client with a diagnosis of myasthenia gravis about home care measures. Which client statement indicates the need for further teaching? 1. "I will rest each afternoon after my walk." 2. "I should cough and deep breathe many times during the day." 3. "I can change the time of my medication on the mornings when I feel strong." 4. "If I get abdominal cramps and diarrhea, I should call my health care provider."

3. "I can change the time of my medication on the mornings when I feel strong." Rationale: The client with myasthenia gravis and the family should be taught information about the disease and its treatment. They should be aware of the side and adverse effects of anticholinesterase medications and corticosteroids and should be taught that timing of anticholinesterase medication is critical. It is important to instruct the client to administer the medication on time to maintain a chemical balance at the neuromuscular junction. If it is not given on time, the client may become too weak to even swallow. Resting after a walk, coughing and deep breathing many times during the day, and calling the primary health care provider when experiencing abdominal cramps and diarrhea indicate a correct understanding of home care instructions to maintain health with this neurological degenerative disease.

The nurse is providing instructions to an adolescent prescribed phenytoin for the control of seizures. Which statement by the adolescent indicates a need for further teaching regarding the medication? 1. "The medication may cause acne or oily skin." 2. "Drinking alcohol may affect the medication." 3. "If my gums become sore and swollen, I need to stop the medication." 4. "Birth control pills may not be effective when I take this medication."

3. "If my gums become sore and swollen, I need to stop the medication." Rationale: The adolescent should not stop taking antiseizure medications suddenly or without discussing it with a primary health care provider (PHCP) or nurse. Acne or oily skin may be a problem for the adolescent, and the adolescent is advised to call a PHCP for skin problems. Alcohol will lower the seizure threshold, and it is best to avoid its use. Birth control pills may be less effective when the client is taking antiseizure medication.

A client is experiencing impotence after taking guanfacine. The client states, "I would sooner have a stroke than keep living with the effects of this medication." What is the most appropriate response by the nurse? 1. "I can understand completely." 2. "You wouldn't really want to have a stroke." 3. "You are concerned about the effects of your medication." 4. "The primary health care provider should change your prescription."

3. "You are concerned about the effects of your medication." Rationale: Reflection of the client's own comment lets the client know that the nurse hears the concern without judging. The nurse cannot understand what the client is experiencing. To tell the client "you wouldn't really want to have a stroke" is confrontational and unsupportive. The client's prescription may need to be changed, but from the options provided this is not the most appropriate response.

The nurse is conducting home visits with a head-injured client with residual cognitive deficits. The client has problems with memory, has a shortened attention span, is easily distracted, and processes information slowly. The nurse plans to talk with the primary health care provider about referring the client to which professional? 1. A psychologist 2. A social worker 3. A neuropsychologist 4. A vocational rehabilitation specialist

3. A neuropsychologist Rationale: Clients with cognitive deficits after head injury may benefit from referral to a neuropsychologist who specializes in evaluating and treating cognitive problems. The neuropsychologist plans an individual program of therapy and initiates counseling to help the client reach maximal potential. The neuropsychologist works in collaboration with other disciplines that are involved in the client's care and rehabilitation. The remaining options are incorrect because these health care workers do not specialize in evaluating and treating cognitive problems.

Dantrolene sodium has been administered to a client with a spinal cord injury. The nurse determines that the client is experiencing a side or adverse effect of the medication if which is noted? 1. Dizziness 2. Drowsiness 3. Abdominal pain 4. Lightheadedness

3. Abdominal pain Rationale: Dantrium is hepatotoxic. The nurse observes for indications of liver dysfunction, which include jaundice, abdominal pain, and malaise. The nurse notifies the primary health care provider if these occur. The signs and symptoms in the remaining options are expected side effects due to the central nervous system-depressant effects of the medication.

A client who suffered a severe head injury has had vigorous treatment to control cerebral edema. Brain death has been determined. The nurse prepares to carry out which measure to maintain viability of the kidneys before organ donation? 1. Assessing lung sounds 2. Monitoring temperature 3. Administering intravenous (IV) fluids 4. Performing range-of-motion exercises to the extremities

3. Administering intravenous (IV) fluids Rationale: Perfusion to the kidney is affected by blood pressure, which in turn is affected by blood vessel tone and fluid volume. Therefore, the client who was previously dehydrated with medications to control intracranial pressure is now in need of rehydration to maintain perfusion to the kidneys. The nurse should prepare to infuse IV fluids as prescribed and continue to monitor urine output. The remaining options will not maintain viability of the kidneys.

The nurse is assisting in the care of a client who is being evaluated for possible myasthenia gravis. The primary health care provider gives a test dose of edrophonium. Evaluation of the results indicates that the test is positive. Which would be the expected response noted by the nurse? 1. Joint pain for the next 15 minutes 2. An immediate increase in blood pressure 3. An increase in muscle strength within 1 to 3 minutes 4. Feelings of faintness or dizziness for 5 to 10 minutes

3. An increase in muscle strength within 1 to 3 minutes Rationale: Edrophonium is a short-acting acetylcholinesterase inhibitor used to diagnose myasthenia gravis. An increase in muscle strength should be seen in 1 to 3 minutes following the test dose if the client does have the disease. If no response occurs, another dose is given over the next 2 minutes and muscle strength is tested again. If no increase in muscle strength occurs with this higher dose, the muscle weakness is not caused by myasthenia gravis. Clients who receive injections of this medication commonly demonstrate a drop in blood pressure, feel faint and dizzy, and are flushed.

The nurse overhears a neurologist saying that a client has an aneurysm located in the circle of Willis. The nurse understands that which blood vessels are part of the circle of Willis? Select all that apply. 1. Basilar artery 2. Vertebral artery 3. Anterior cerebral artery 4. Internal carotid arteries 5. Posterior cerebral artery

3. Anterior cerebral artery 4. Internal carotid arteries 5. Posterior cerebral artery Rationale: The circle of Willis is a ring of blood vessels located at the base of the brain. It is referred to as the anterior circulation to the brain and is composed of the anterior and middle cerebral arteries, posterior cerebral arteries, posterior communicating arteries, internal carotid arteries, and anterior communicating branches. The basilar artery and vertebral artery are not part of the circle of Willis. Rather, they are part of the vertebral-basilar system, which is known as the posterior circulation to the brain. Other parts of the posterior circulation are the posterior inferior cerebellar artery and the spinal arteries

A nursing student is caring for a client with a stroke (brain attack) who is experiencing unilateral neglect. The nurse would intervene if the student plans to use which strategy to help the client adapt to this deficit? 1. Telling the client to scan the environment 2. Placing the bedside articles on the affected side 3. Approaching the client from the unaffected side 4. Moving the commode and chair to the affected side

3. Approaching the client from the unaffected side Rationale: Unilateral neglect is an unawareness of the paralyzed side of the body, which increases a client's risk for injury. The nurse's role is to refocus the client's attention to the affected side. The nurse moves personal care items and belongings to the affected side, as well as the bedside chair and commode. The nurse teaches the client to scan the environment so as to become aware of the affected half of the body. The nurse approaches the client from the affected side to increase awareness further.

The nurse is assisting in the care of a client with myasthenia gravis who is receiving pyridostigmine. Which medication should the nurse plan to have readily available should the client develop cholinergic crisis because of excessive medication dosage? 1. Vitamin K 2. Acetylcysteine 3. Atropine sulfate 4. Protamine sulfate

3. Atropine sulfate Rationale: If the client is in cholinergic crisis, the antidote for the medication would be a medication that is an anticholinergic. Thus, the antidote for cholinergic crisis is atropine sulfate. Vitamin K is the antidote for warfarin. Protamine sulfate is the antidote for heparin, and acetylcysteine is the antidote for acetaminophen.

The nurse is administering an intravenous dose of methocarbamol to a client with multiple sclerosis. For which adverse effect should the nurse monitor? 1. Tachycardia 2. Rapid pulse 3. Bradycardia 4. Hypertension

3. Bradycardia Rationale: Intravenous administration of methocarbamol can cause hypotension and bradycardia. The nurse needs to monitor for these adverse effects. Options 1, 2, and 4 are not effects with administration of this medication.

At 8:00 a.m., a client who has had a stroke (brain attack) was awake and alert with vital signs of temperature 98º F (37.2º C) orally, pulse 80 beats/min, respirations 18 breaths/min, and blood pressure 138/80 mm Hg. At noon, the client is confused and only responsive to tactile stimuli, and vital signs are temperature 99º F (36.7º C) orally, pulse 62 beats/min, respirations 20 breaths/min, and blood pressure 166/72 mm Hg. The nurse should take which action? 1. Reorient the client. 2. Retake the vital signs. 3. Call the primary health care provider (PHCP). 4. Administer an antihypertensive PRN (as needed).

3. Call the primary health care provider (PHCP). Rationale: The important nursing action is to call the PHCP. The deterioration in neurological status, decreasing pulse, and increasing blood pressure with a widening pulse pressure all indicate that the client is experiencing increased intracranial pressure, which requires immediate treatment to prevent further complications and possible death. The nurse should retake the vital signs and reorient the client to surroundings. If the client's blood pressure falls within parameters for PRN antihypertensive medication, the medication also should be administered. However, options 1, 2, and 4 are secondary nursing actions.

The nurse in the neurological unit is caring for a client who was in a motor vehicle crash and sustained a blunt head injury. On assessment of the client, the nurse notes the presence of bloody drainage from the nose. Which nursing action is most appropriate? 1. Insert nasal packing. 2. Document the findings. 3. Contact the primary health care provider (PHCP). 4. Monitor the client's blood pressure and check for signs of increased intracranial pressure.

3. Contact the primary health care provider (PHCP). Rationale: Bloody or clear drainage from either the nasal or the auditory canal after head trauma could indicate a cerebrospinal fluid leak. The appropriate nursing action is to notify the PHCP because this finding requires immediate intervention. The remaining options are inappropriate nursing actions in this situation.

Baclofen is prescribed for the client with multiple sclerosis. The nurse determines that the medication is having the intended effect if which finding is noted in the client? 1. Increased muscle tone 2. Increased range of motion 3. Decreased muscle spasms 4. Decreased local pain and tenderness

3. Decreased muscle spasms Rationale: Baclofen is a skeletal muscle relaxant and acts at the spinal cord level to decrease the frequency and degree of muscle spasms in clients with multiple sclerosis, spinal cord injury, or other diseases. The other options are incorrect.

The nurse is assessing a client with a brainstem injury. In addition to obtaining the client's vital signs and determining the Glasgow Coma Scale score, what priority intervention should the nurse plan to implement? 1. Check cranial nerve functioning. 2. Determine the cause of the accident. 3. Draw blood for arterial blood gas analysis. 4. Perform a pulmonary wedge pressure measurement.

3. Draw blood for arterial blood gas analysis. Rationale: Assessment should be specific to the area of the brain involved. The respiratory center is located in the brainstem. Assessing the respiratory status is the priority for a client with a brainstem injury. The actions in the remaining options are not priorities, although they may be a component in the assessment process, depending on the injury and client condition.

Dantrolene is prescribed for a client with spinal cord injury for discomfort caused by spasticity. Which finding would alert the nurse to a potential adverse effect associated with this medication? 1. Headache 2. Blurred vision 3. Elevated temperature 4. Abdominal distention

3. Elevated temperature Rationale: Dantrolene is a centrally acting muscle relaxant. Malignant hyperthermia is a rare but life-threatening adverse effect that can occur with use of this medication. Therefore, an elevated temperature would alert the nurse to this potential adverse effect.

The nurse is evaluating the neurological signs of a client in spinal shock following spinal cord injury. Which observation indicates that spinal shock persists? 1. Hyperreflexia 2. Positive reflexes 3. Flaccid paralysis 4. Reflex emptying of the bladder

3. Flaccid paralysis Rationale: Resolution of spinal shock is occurring when there is return of reflexes (especially flexors to noxious cutaneous stimuli), a state of hyper-reflexia rather than flaccidity, and reflex emptying of the bladder.

The client with a head injury opens eyes to sound, has no verbal response, and localizes to painful stimuli when applied to each extremity. How should the nurse document the Glasgow Coma Scale (GCS) score? 1. GCS = 3 2. GCS = 6 3. GCS = 9 4. GCS = 11

3. GCS = 9 Rationale: The GCS is a method for assessing neurological status. The highest possible GCS score is 15. A score lower than 8 indicates that coma is present. Motor response points are as follows: Obeys a simple response = 6; Localizes painful stimuli = 5; Normal flexion (withdrawal) = 4; Abnormal flexion (decorticate posturing) = 3; Extensor response (decerebrate posturing) = 2; No motor response to pain = 1. Verbal response points are as follows: Oriented = 5; Confused conversation = 4; Inappropriate words = 3; Responds with incomprehensible sounds = 2; No verbal response = 1. Eye opening points are as follows: Spontaneous = 4; In response to sound = 3; In response to pain = 2; No response, even to painful stimuli = 1. Using the GCS, a score of 3 is given when the client opens the eyes to sound. Localization to pain is scored as 5. When there is no verbal response, the score is 1. The total score is then equal to 9.

A client brought to the emergency department had a seizure 1 hour ago. Family members were present during the episode and reported that the client's jaw was moving as though grinding food. In helping to determine the origin of this seizure, what should the nurse include in the client's assessment? 1. Loss of consciousness 2. Presence of diaphoresis 3. History of prior trauma 4. Rotating eye movements

3. History of prior trauma Rationale: Seizures that originate with specific motor phenomena are considered focal and are indicative of a focal structural lesion in the brain, often caused by trauma, infection, or medication consumption. The remaining options address signs, rather than an origin of the seizure.

A client with a traumatic closed head injury shows signs of secondary brain injury. What are some manifestations of secondary brain injury? Select all that apply. 1. Fever 2. Seizures 3. Hypoxia 4. Ischemia 5. Hypotension 6. Increased intracranial pressure (ICP)

3. Hypoxia 4. Ischemia 5. Hypotension 6. Increased intracranial pressure (ICP) Rationale: Secondary brain injury can occur several hours to days after the initial brain injury and is a major concern when managing brain trauma. Nursing management of the client with an acute intracranial problem must include management of secondary injury. Manifestations of secondary injury include hypoxia, ischemia, hypotension, and increased ICP that follows primary injury. It does not include fever or seizures.

A client with multiple sclerosis is experiencing muscle weakness, spasticity, and an ataxic gait. On the basis of this information, the nurse should include which client problem in the plan of care? 1. Inability to care for self 2. Interruption in skin integrity 3. Interruption in physical mobility 4. Inability to perform daily activities

3. Interruption in physical mobility Rationale: Multiple sclerosis is a chronic, nonprogressive, noncontagious degenerative disease of the central nervous system characterized by demyelination of the neurons. Interruption in physical mobility is most appropriate for the client with multiple sclerosis experiencing muscle weakness, spasticity, and ataxic gait. The remaining options are not related to the data in the question.

A client with a history of myasthenia gravis presents at a clinic with bilateral ptosis and is drooling, and myasthenic crisis is suspected. The nurse assesses the client for which precipitating factor? 1. Getting too little exercise 2. Taking excess medication 3. Omitting doses of medication 4. Increasing intake of fatty foods

3. Omitting doses of medication Rationale: Myasthenic crisis often is caused by undermedication and responds to the administration of cholinergic medications. Cholinergic crisis (the opposite problem) is caused by excess medication and responds to withholding of medications. Too little exercise and excessive fatty food intake are incorrect. Overexertion and overeating possibly could trigger myasthenic crisis.

The nurse is preparing for the admission to the unit of a client with a diagnosis of seizures and asks the nursing student to institute full seizure precautions. Which item if noted in the client's room would need to be removed and warrants the need to review seizure precautions with the student? 1. Oxygen source 2. Suction machine 3. Padded tongue blade 4. Padding for the side rails

3. Padded tongue blade Rationale: Full seizure precautions include bed rest with padded side rails in a raised position, a suction machine at the bedside, having diazepam or lorazepam available, and providing an oxygen source. Objects such as tongue blades are contraindicated and should never be placed in the client's mouth during a seizure.

The client with a spinal cord injury at the level of T4 is experiencing a severe throbbing headache with a blood pressure of 180/100 mm Hg. What is the priority nursing intervention? 1. Notify the neurologist. 2. Loosen tight clothing on the client. 3. Place the client in a sitting position. 4. Check the urinary catheter tubing for kinks or obstruction.

3. Place the client in a sitting position. Rationale: The client is demonstrating clinical manifestations of autonomic dysreflexia, which is a neurological emergency. The first priority is to place the client in a sitting position to prevent hypertensive stroke. Loosening tight clothing and checking the urinary catheter can then be done, and the neurologist can be notified once initial interventions are done.

A client with trigeminal neuralgia tells the nurse that acetaminophen is taken daily for the relief of generalized discomfort. Which laboratory value would indicate toxicity associated with the medication? 1. Sodium level of 140 mEq/L (140 mmol/L) 2. Platelet count of 400,000 mm3 (400 × 109/L) 3. Prothrombin time of 12 seconds (12 seconds) 4. Direct bilirubin level of 2 mg/dL (34 mcmol/L)

4. Direct bilirubin level of 2 mg/dL (34 mcmol/L) Rationale: In adults, overdose of acetaminophen causes liver damage. The correct option is an indicator of liver function and is the only option that indicates an abnormal laboratory value. The normal direct bilirubin level is 0.1 to 0.3 mm3 (150 to 400 × 109/L).

A client with Guillain-Barré syndrome has ascending paralysis and is intubated and receiving mechanical ventilation. Which strategy should the nurse incorporate in the plan of care to help the client cope with this illness? 1. Giving client full control over care decisions and restricting visitors 2. Providing positive feedback and encouraging active range of motion 3. Providing information, giving positive feedback, and encouraging relaxation 4. Providing intravenously administered sedatives, reducing distractions, and limiting visitors

3. Providing information, giving positive feedback, and encouraging relaxation Rationale: The client with Guillain-Barré syndrome experiences fear and anxiety from the ascending paralysis and sudden onset of the disorder. The nurse can alleviate these fears by providing accurate information about the client's condition, giving expert care and positive feedback to the client, and encouraging relaxation and distraction. The family can become involved with selected care activities and provide diversion for the client as well.

The nurse is caring for a client with impaired mobility that occurred as a result of a stroke. The client has right-sided arm and leg weakness. Which assistive device should the nurse suggest that the client use to provide the best stability for ambulating? 1. Walker 2. Crutches 3. Quad cane 4. Single straight-legged cane

3. Quad cane Rationale: Crutches and a walker can be difficult to maneuver for a client with weakness on 1 side. A cane is better suited for the client with weakness of the arm and leg on 1 side; however, the quad cane would provide the most stability because of the structure of the cane and because a quad cane has 4 legs

The nurse is admitting to the hospital a client with a diagnosis of Guillain-Barré syndrome. The nurse knows that if the disease is severe, the client will be at risk for which acid-base imbalance? 1. Metabolic acidosis 2. Metabolic alkalosis 3. Respiratory acidosis 4. Respiratory alkalosis

3. Respiratory acidosis Rationale: Guillain-Barré is a neuromuscular disorder in which the client may experience weakening or paralysis of the muscles used for respiration. This could cause the client to retain carbon dioxide, leading to respiratory acidosis and ventilatory failure as the paralysis develops. Therefore, the remaining options are incorrect.

A client is taking the prescribed dose of phenytoin to control seizures. Results of a phenytoin blood level study reveal a level of 35 mcg/mL (140 mcmol/L). Which finding would be expected as a result of this laboratory result? 1. Hypotension 2. Tachycardia 3. Slurred speech 4. No abnormal finding

3. Slurred speech Rationale: The therapeutic phenytoin level is 10 to 20 mcg/mL (40 to 79 mcmol/L). At a level higher than 20 mcg/mL, involuntary movements of the eyeballs (nystagmus) occur. At a level higher than 30 mcg/mL (120 mcmol/L), ataxia and slurred speech occur.

The nurse is reviewing the medical records of a client admitted to the nursing unit with a diagnosis of a thrombotic brain attack (stroke). The nurse would expect to note that which is documented in the assessment data section of the record? 1. Sudden loss of consciousness occurred. 2. Signs and symptoms occurred suddenly. 3. The client experienced paresthesias a few days before admission to the hospital. 4. The client complained of a severe headache, which was followed by sudden onset of paralysis.

3. The client experienced paresthesias a few days before admission to the hospital. Rationale: Cerebral thrombosis does not occur suddenly. In the few hours or days preceding a thrombotic brain attack (stroke), the client may experience a transient loss of speech, hemiplegia, or paresthesias on 1 side of the body. Signs and symptoms of thrombotic brain attack (stroke) vary but may include dizziness, cognitive changes, or seizures. Headache is rare, but some clients with stroke (brain attack) experience signs and symptoms similar to those of cerebral embolism or intracranial hemorrhage.

The home health nurse visits a client who is taking phenytoin for control of seizures. During the assessment, the nurse notes that the client is taking birth control pills. Which information should the nurse include in the teaching plan? 1. Pregnancy must be avoided while taking phenytoin. 2. The client may stop the medication if it is causing severe gastrointestinal effects. 3. There is the potential of decreased effectiveness of birth control pills while taking phenytoin. 4. There is the increased risk of thrombophlebitis while taking phenytoin and birth control pills together.

3. There is the potential of decreased effectiveness of birth control pills while taking phenytoin. Rationale: Phenytoin enhances the rate of estrogen metabolism, which can decrease the effectiveness of some birth control pills. Options 1, 2, and 4 are inappropriate instructions. Pregnancy does not need to be "avoided" while taking phenytoin; however, because phenytoin may cause some risk to the fetus (Pregnancy Category D medication), consultation with the primary health care provider should be done if pregnancy is considered. Telling a client that there is an increased risk of thrombophlebitis is incorrect and inappropriate and could cause anxiety in the client. A client should not be instructed to stop antiseizure medication.

A client is brought to the emergency department by emergency medical services (EMS) after being hit by a car. The name of the client is unknown, and the client has sustained a severe head injury and multiple fractures and is unconscious. An emergency craniotomy is required. Regarding informed consent for the surgical procedure, which is the best action? 1. Obtain a court order for the surgical procedure. 2. Ask the EMS team to sign the informed consent. 3. Transport the victim to the operating room for surgery. 4. Call the police to identify the client and locate the family.

3. Transport the victim to the operating room for surgery. Rationale: In general, there are two situations in which informed consent of an adult client is not needed. One is when an emergency is present and delaying treatment for the purpose of obtaining informed consent would result in injury or death to the client. The second is when the client waives the right to give informed consent. Option 1 will delay emergency treatment, and option 2 is inappropriate. Although option 4 may be pursued, it is not the best action because it delays necessary emergency treatment.

A client has suffered a head injury affecting the occipital lobe of the brain. What is the focus of the nurse's immediate assessment? 1. Taste 2. Smell 3. Vision 4. Hearing

3. Vision Rationale: The occipital lobe is responsible for reception of vision and contains visual association areas. This area of the brain helps the individual to visually recognize and understand the surroundings. The other senses listed are not a function of the occipital lobe.

A patient comes into the ED with a suspected acute CVA. As the nurse, you anticipate which medication to be given to this client? 1. Baby aspirin 2. Plavix 3. tPA 4. Coumadin

3. tPA Rationale: Clotbuster! Used for acute stroke if not contraindicated by a head injury. Needs to be used within first 3-4 hours after CVA symptoms. Baby aspirin and Plavix are preventative medications, and Coumadin is used before discharge, but takes too long to get into the system to be used initially after stroke symptoms are present.

The nurse is performing an assessment on a client with a diagnosis of thrombotic stroke (brain attack). Which assessment question would elicit data specific to this type of stroke? 1. "Have you had any headaches in the past few days?" 2. "Have you recently been having difficulty with seeing at nighttime?" 3. "Have you had any sudden episodes of passing out in the past few days?" 4. "Have you had any numbness or tingling or paralysis-type feelings in any of your extremities recently?"

4. "Have you had any numbness or tingling or paralysis-type feelings in any of your extremities recently?" Rationale: Cerebral thrombosis (thrombotic stroke) does not occur suddenly. In the few days or hours preceding the thrombotic stroke, the client may experience a transient loss of speech, hemiparesis, or paresthesias on 1 side of the body. Signs and symptoms of this type of stroke vary but may also include dizziness, cognitive changes, or seizures. Headache is rare, but some clients with stroke experience signs and symptoms similar to those of cerebral embolism or intracranial hemorrhage. The client does not complain of difficulty with night vision as part of this clinical problem. In addition, most clients do not have repeated episodes of loss of consciousness.

The nurse has given suggestions to a client with trigeminal neuralgia about strategies to minimize episodes of pain. The nurse determines that the client needs further teaching if the client makes which statement? 1. "I will wash my face with cotton pads." 2. "I'll have to start chewing on my unaffected side." 3. "I should rinse my mouth if toothbrushing is painful." 4. "I'll try to eat my food either very warm or very cold."

4. "I'll try to eat my food either very warm or very cold." Rationale: Facial pain can be minimized by using cotton pads to wash the face and using room temperature water. The client should chew on the unaffected side of the mouth, eat a soft diet, and take in foods and beverages at room temperature. If brushing the teeth triggers pain, an oral rinse after meals may be helpful instead.

The home care nurse is preparing to visit a client with a diagnosis of trigeminal neuralgia (tic douloureux). When performing the assessment, the nurse should plan to ask the client which question to elicit the most specific information regarding this disorder? 1. "Do you have any visual problems?" 2. "Are you having any problems hearing?" 3. "Do you have any tingling in the face region?" 4. "Is the pain experienced a stabbing type of pain?"

4. "Is the pain experienced a stabbing type of pain?" Rationale: Trigeminal neuralgia is characterized by spasms of pain that start suddenly and last for seconds to minutes. The pain often is characterized as stabbing or as similar to an electric shock. It is accompanied by spasms of facial muscles that cause twitching of parts of the face or mouth, or closure of the eye. The remaining options do not elicit data specifically related to this disorder.

The nurse is caring for a client diagnosed with trigeminal neuralgia. The client asks the nurse, "Why do I have so much pain?" Which is the appropriate response by the nurse? 1. "It's a local reaction to nasal stuffiness." 2. "It's due to a hypoglycemic effect on the cranial nerve." 3. "Release of catecholamines with infection or stress leads to the pain." 4. "Pain is due to stimulation of the affected nerve by pressure and temperature."

4. "Pain is due to stimulation of the affected nerve by pressure and temperature." Rationale: The paroxysms of pain that accompany this neuralgia are triggered by stimulation of the terminal branches of the trigeminal nerve. Symptoms can be triggered by pressure from washing the face, brushing the teeth, shaving, eating, or drinking. Symptoms also can be triggered by thermal stimuli, such as a draft of cold air. The remaining options are incorrect.

The nurse has instructed the family of a client with stroke (brain attack) who has homonymous hemianopsia about measures to help the client overcome the deficit. Which statement suggests that the family understands the measures to use when caring for the client? 1. "We need to discourage him from wearing eyeglasses." 2. "We need to place objects in his impaired field of vision." 3. "We need to approach him from the impaired field of vision." 4. "We need to remind him to turn his head to scan the lost visual field."

4. "We need to remind him to turn his head to scan the lost visual field." Rationale: Homonymous hemianopsia is loss of half of the visual field. The client with homonymous hemianopsia should have objects placed in the intact field of vision, and the nurse also should approach the client from the intact side. The nurse instructs the client to scan the environment to overcome the visual deficit and does client teaching from within the intact field of vision. The nurse encourages the use of personal eyeglasses, if they are available.

A client with myasthenia gravis has difficulty chewing and has received a prescription for pyridostigmine. The nurse should check to see that the client takes the medication at what time? 1. With meals 2. Between meals 3. Just after meals 4. 30 minutes before meals

4. 30 minutes before meals Rationale: Pyridostigmine is a cholinergic medication used to increase muscle strength in the client with myasthenia gravis. For the client who has difficulty chewing, the medication should be administered 30 minutes before meals to enhance the client's ability to eat. The times noted in the remaining options will not be helpful to the client

The nurse is planning care for the client with a neurogenic bladder caused by multiple sclerosis. The nurse plans for fluid administration of at least 2000 mL/day. Which plan would be most helpful to this client? 1. 400 to 500 mL with each meal and 500 to 600 mL in the evening before bedtime 2. 400 to 500 mL with each meal and additional fluids in the morning but not after midday 3. 400 to 500 mL with each meal, with all extra fluid concentrated in the afternoon and evening 4. 400 to 500 mL with each meal and 200 to 250 mL at midmorning, midafternoon, and late afternoon

4. 400 to 500 mL with each meal and 200 to 250 mL at midmorning, midafternoon, and late afternoon Rationale: Spacing fluid intake over the day helps the client with a neurogenic bladder to establish regular times for successful voiding. Omitting intake after the evening meal minimizes incontinence or the need to empty the bladder during the night.

A client with myasthenia gravis has become increasingly weaker. The primary health care provider prepares to identify whether the client is reacting to an overdose of the medication (cholinergic crisis) or an increasing severity of the disease (myasthenic crisis). An injection of edrophonium is administered. Which finding would indicate that the client is in cholinergic crisis? 1. No change in the condition 2. Complaints of muscle spasms 3. An improvement of the weakness 4. A temporary worsening of the condition

4. A temporary worsening of the condition Rationale: An edrophonium injection makes the client in cholinergic crisis temporarily worse. An improvement in the weakness indicates myasthenia crisis. Muscle spasms are not associated with this test.

The nurse is caring for a client in the emergency department who has been diagnosed with Bell's palsy. The client has been taking acetaminophen, and acetaminophen overdose is suspected. Which antidote should the nurse prepare for administration if prescribed? 1. Pentostatin 2. Auranofin 3. Fludarabine 4. Acetylcysteine

4. Acetylcysteine Rationale: The antidote for acetaminophen is acetylcysteine. The normal therapeutic serum level of acetaminophen is 10 to 20 mcg/mL. A toxic level is higher than 50 mcg/mL, and levels higher than 200 mcg/mL 4 hours after ingestion indicates that there is risk for liver damage. Auranofin is a gold preparation that may be used to treat rheumatoid arthritis. Pentostatin and fludarabine are antineoplastic agents.

A client with a spinal cord injury becomes angry and belligerent whenever the nurse tries to administer care. The nurse should perform which action? 1. Ask the family to deliver the care. 2. Leave the client alone until ready to participate. 3. Advise the client that rehabilitation progresses more quickly with cooperation. 4. Acknowledge the client's anger and continue to encourage participation in care.

4. Acknowledge the client's anger and continue to encourage participation in care. Rationale: Adjusting to paralysis is physically and psychosocially difficult for the client and family. The nurse recognizes that the client goes through the grieving process in adjusting to the loss and may move back and forth among the stages of grief. The nurse acknowledges the client's feelings while continuing to meet the client's physical needs and encouraging independence. The family also is in crisis and needs the nurse's support and should not be relied on to provide care. The nurse cannot simply neglect the client until the client is ready to participate. Option 3 represents a factual but noncaring approach to the client and is not therapeutic

A client with a subarachnoid hemorrhage needs to have surgery delayed until a stable clinical condition is achieved. The nurse prepares to administer which medication as prescribed to prevent clot breakdown and dissolution? 1. Alteplase 2. Heparin sodium 3. Warfarin sodium 4. Aminocaproic acid

4. Aminocaproic acid Rationale: Aminocaproic acid is an antifibrinolytic agent that prevents clot breakdown or dissolution. It is commonly prescribed after subarachnoid hemorrhage if surgery is delayed or contraindicated, to prevent further hemorrhage. Alteplase is a fibrinolytic that actively breaks down clots. Warfarin sodium and heparin sodium are anticoagulants that interfere with propagation or growth of a clot.

A client is suspected of having myasthenia gravis. Edrophonium is administered intravenously to determine the diagnosis. Which indicates that the client may have myasthenia gravis? 1. Joint pain following administration of the medication 2. Feelings of faintness, dizziness, hypotension, and signs of flushing in the client 3. A decrease in muscle strength within 30 to 60 seconds following administration of the medication 4. An increase in muscle strength within 30 to 60 seconds following administration of the medication

4. An increase in muscle strength within 30 to 60 seconds following administration of the medication Rationale: Edrophonium is a short-acting acetylcholinesterase inhibitor used as a diagnostic agent. When a client has suspected myasthenia gravis, the health care provider will administer an edrophonium test. When a dose is administered intravenously, an increase in muscle strength should be seen in 30 to 60 seconds. If no response occurs, another dose of edrophonium is given over the next 2 minutes, and muscle strength is tested again. If no increase in muscle strength occurs with this higher dose, the muscle weakness is not caused by myasthenia gravis. Clients receiving injections of this medication commonly demonstrate a drop in blood pressure, feel faint and dizzy, and are flushed.

The nurse is creating a plan of care for a client with a stroke (brain attack) who has global aphasia. The nurse should incorporate communication strategies into the plan of care because of which expected characteristic of the client's speech? 1. Intact 2. Rambling 3. Characterized by literal paraphasia 4. Associated with poor comprehension

4. Associated with poor comprehension Rationale: Global aphasia is a condition in which the affected person has few language skills as a result of extensive damage to the left hemisphere. The speech is nonfluent and is associated with poor comprehension and limited ability to name objects or repeat words. The client with conduction aphasia has difficulty repeating words spoken by another, and speech is characterized by literal paraphasia with intact comprehension. The client with Wernicke's aphasia may exhibit a rambling type of speech.

A client who has a spinal cord injury that resulted in paraplegia experiences a sudden onset of severe headache and nausea. The client is diaphoretic with piloerection and has flushing of the skin. The client's systolic blood pressure (BP) is 210 mm Hg. What should the nurse immediately suspect? 1. Return of spinal shock 2. Malignant hypertension 3. Impending brain attack (stroke) 4. Autonomic dysreflexia (hyperreflexia)

4. Autonomic dysreflexia (hyperreflexia) Rationale: Autonomic dysreflexia (hyperreflexia) results from sudden strong discharge of the sympathetic nervous system in response to a noxious stimulus. Signs and symptoms include pounding headache, nausea, nasal stuffiness, flushed skin, piloerection, and diaphoresis. Severe hypertension can occur, with a systolic BP rising potentially as high as 300 mm Hg. It often is triggered by thermal or mechanical events such as a kinking of catheter tubing, constipation, urinary tract infection, or any variety of cutaneous stimuli. The nurse must recognize this situation immediately and take corrective action to remove the stimulus. If untreated, this medical emergency could result in stroke, status epilepticus, or possibly death.

The nurse is providing instructions to the client with trigeminal neuralgia regarding measures to take to prevent the episodes of pain. Which should the nurse instruct the client to do? 1. Prevent stressful situations. 2. Avoid activities that may cause fatigue. 3. Avoid contact with people with an infection. 4. Avoid activities that may cause pressure near the face.

4. Avoid activities that may cause pressure near the face. Rationale: The pain that accompanies trigeminal neuralgia is triggered by stimulation of the trigeminal nerve. Symptoms can be triggered by pressure such as from washing the face, brushing the teeth, shaving, eating, or drinking. Symptoms also can be triggered by stimulation by a draft or cold air. The remaining options are not associated with triggering episodes of pain.

You are the nurse taking care of a client who has suffered a spinal cord injury, which of the following vital signs indicate the client has developed spinal shock? 1. BP 178/90, heart rate 110, temp 99.5 2. BP 80/50, heart rate 120, temp 98.3 3. BP 200/78, heart rate 52, temp 100.1 4. BP 76/48, heart rate 50, temp 97.0

4. BP 76/48, heart rate 50, temp 97.0 Rationale: Spinal shock, hypotensive and bradycardia. Same-SS

The nurse prepares to teach a client with subarachnoid hemorrhage about the effects of nimodipine. The nurse plans to explain which information about the type and action of this medication? 1. Vasodilator that has an affinity for cerebral blood vessels 2. Beta-adrenergic blocker that will decrease blood pressure 3. Diuretic that will decrease blood pressure by decreasing fluid volume 4. Calcium channel blocker that will decrease spasm in cerebral blood vessels

4. Calcium channel blocker that will decrease spasm in cerebral blood vessels Rationale: Nimodipine is a calcium channel-blocking agent that has an affinity for cerebral blood vessels. It is used to prevent or control vasospasm in cerebral blood vessels, thereby reducing the chance for rebleeding. It is typically prescribed for 3 weeks' duration.

The nurse is assessing the adaptation of a client to changes in functional status after a stroke (brain attack). Which observation indicates to the nurse that the client is adapting most successfully? 1. Gets angry with family if they interrupt a task 2. Experiences bouts of depression and irritability 3. Has difficulty with using modified feeding utensils 4. Consistently uses adaptive equipment in dressing self

4. Consistently uses adaptive equipment in dressing self Rationale: Clients are evaluated as coping successfully with lifestyle changes after a stroke if they make appropriate lifestyle alterations, use the assistance of others, and have appropriate social interactions. Options 1 and 2 are not adaptive behaviors; option 3 indicates a not yet successful attempt to adapt.

The nurse is reviewing the results of a test on a sample drawn from a child who is receiving carbamazepine for the control of seizures. The results indicate a serum carbamazepine level of 10 mcg/mL (42.33 mmol/L). The nurse analyzes the results and anticipates that the primary health care provider (PHCP) will note which prescription? 1. Discontinuation of the medication 2. A decreased dose of the medication 3. An increased dose of the medication 4. Continuation of the presently prescribed dosage

4. Continuation of the presently prescribed dosage Rationale: When carbamazepine is administered, blood levels need to be tested periodically to check for the child's absorption of the medication. The amount of the medication prescribed is based on the blood level achieved. Carbamazepine's therapeutic serum range is 6 to 12 mcg/mL (34 to 51 mmol/L). Therefore, the nurse anticipates that the PHCP will continue the presently prescribed dosage.

The nurse is assigned to care for a client with multiple sclerosis who is receiving an intravenous dose of methocarbamol. The nurse monitors the client knowing that which is an expected side effect? 1. Insomnia 2. Excitability 3. Hypertension 4. Dark green-colored urine

4. Dark green-colored urine Rationale: Methocarbamol is a skeletal muscle relaxant. It may cause the urine to turn a brown, black, or dark green color, and the client needs to be told that this is a harmless effect. This medication can cause hypotension. Drowsiness and dizziness can also occur. Therefore, the remaining options are incorrect.

The nurse is preparing for the admission of a client with a suspected diagnosis of Guillain-Barré syndrome. When the client arrives at the nursing unit, the nurse reviews the primary health care provider's documentation. The nurse expects to note documentation of which hallmark clinical manifestation of this syndrome? 1. Multifocal seizures 2. Altered level of consciousness 3. Abrupt onset of a fever and headache 4. Development of progressive muscle weakness

4. Development of progressive muscle weakness Rationale: A hallmark clinical manifestation of Guillain-Barré syndrome is progressive muscle weakness that develops rapidly. Seizures are not normally associated with this disorder. The client does not have symptoms such as a fever or headache. Cerebral function, level of consciousness, and pupillary responses are normal.

The nurse is teaching a client hospitalized with a seizure disorder and the client's spouse about safety precautions after discharge. The nurse determines that the client needs further teaching if the client states an intention to take which action? 1. Refrain from smoking alone. 2. Take all prescribed medications on time. 3. Have the spouse nearby when showering. 4. Drink alcohol in small amounts and only on weekends.

4. Drink alcohol in small amounts and only on weekends. Rationale: The client should avoid the intake of alcohol. Alcohol could interact with the client's seizure medications, or it could precipitate seizure activity. The client should take all medications on time to avoid decreases in therapeutic medication levels, which could precipitate seizures. The client should not bathe in the shower or tub without someone nearby and should not smoke alone, to minimize the risk of injury if a seizure occurs.

The nurse is admitting a client with Guillain-Barré syndrome to the nursing unit. The client has complaints of inability to move both legs and reports a tingling sensation above the waistline. Knowing the complications of the disorder, the nurse should bring which most essential items into the client's room? 1. Nebulizer and pulse oximeter 2. Blood pressure cuff and flashlight 3. Nasal cannula and incentive spirometer 4. Electrocardiographic monitoring electrodes and intubation tray

4. Electrocardiographic monitoring electrodes and intubation tray Rationale: The client with Guillain-Barré syndrome is at risk for respiratory failure because of ascending paralysis. An intubation tray should be available for use. Another complication of this syndrome is cardiac dysrhythmias, which necessitates the use of electrocardiographic monitoring. Because the client is immobilized, the nurse should assess for deep vein thrombosis and pulmonary embolism routinely. Although items in the incorrect options may be used in care, they are not the most essential items from the options provided.

The nurse is caring for a client in the emergency department who has sustained a head injury. The client momentarily lost consciousness at the time of the injury and then regained it. The client now has lost consciousness again. The nurse takes quick action, knowing that this sequence is compatible with which most likely condition? 1. Concussion 2. Skull fracture 3. Subdural hematoma 4. Epidural hematoma

4. Epidural hematoma Rationale: The changes in neurological signs from an epidural hematoma begin with loss of consciousness as arterial blood collects in the epidural space and exerts pressure. The client regains consciousness as the cerebrospinal fluid is reabsorbed rapidly to compensate for the rising intracranial pressure. As the compensatory mechanisms fail, even small amounts of additional blood cause the intracranial pressure to rise rapidly, and the client's neurological status deteriorates quickly.

A client recovering from a head injury is participating in care. The nurse determines that the client understands measures to prevent elevations in intracranial pressure if the nurse observes the client doing which activity? 1. Blowing the nose 2. Isometric exercises 3. Coughing vigorously 4. Exhaling during repositioning

4. Exhaling during repositioning Rationale: Activities that increase intrathoracic and intraabdominal pressures cause an indirect elevation of the intracranial pressure. Some of these activities include isometric exercises, Valsalva's maneuver, coughing, sneezing, and blowing the nose. Exhaling during activities such as repositioning or pulling up in bed opens the glottis, which prevents intrathoracic pressure from rising.

A client has clear fluid leaking from the nose following a basilar skull fracture. Which finding would alert the nurse that cerebrospinal fluid is present? 1. Fluid is clear and tests negative for glucose. 2. Fluid is grossly bloody in appearance and has a pH of 6. 3. Fluid clumps together on the dressing and has a pH of 7. 4. Fluid separates into concentric rings and tests positive for glucose.

4. Fluid separates into concentric rings and tests positive for glucose. Rationale: Leakage of cerebrospinal fluid (CSF) from the ears or nose may accompany basilar skull fracture. CSF can be distinguished from other body fluids, because the drainage will separate into bloody and yellow concentric rings on dressing material, called a halo sign. The fluid also tests positive for glucose.

A client who is taking phenytoin for a seizure disorder is being admitted to the hospital because of an increase in seizure activity. The client reports severe vomiting for the last 24 hours and an inability to take phenytoin during that time. The nurse anticipates that the primary health care provider will most likely prescribe which medication? 1. Clonazepam 2. Phenobarbital 3. Carbamazepine 4. Fosphenytoin sodium

4. Fosphenytoin sodium Rationale: Fosphenytoin sodium is used for short-term parenteral (intravenous) infusion. A client who is not tolerating medications orally and has a seizure disorder would need an anticonvulsant administered by the parenteral route. Phenobarbital is an antiseizure medication that is given orally or parenterally. However, the medication of choice in this case would be fosphenytoin since its use is short term. Carbamazepine and clonazepam usually are administered orally.

The nurse is caring for a client who has just been admitted to the hospital with a diagnosis of a hemorrhagic stroke. The nurse should place the client in which position? 1. Prone 2. Supine 3. Semi-Fowler's with the hip and the neck flexed 4. Head of the bed elevated 30 degrees with the head in midline position

4. Head of the bed elevated 30 degrees with the head in midline position Rationale: The primary health care provider's prescriptions are always followed with regard to positioning the client after stroke. Clients with hemorrhagic stroke usually have the head of the bed elevated to 30 degrees to reduce intracranial pressure that can occur from the hemorrhage. The head should be in a midline, neutral position to facilitate venous drainage from the brain. Extreme hip and neck flexion should be avoided to prevent an increase in intrathoracic pressure and to promote venous drainage from the brain. For clients with ischemic stroke, the head of the bed usually is kept flat to ensure adequate blood flow and thus oxygenation to the brain. Prone, supine, and hip and neck flexion are incorrect positions for clients with hemorrhagic stroke.

The nurse is caring for the client who suffered a spinal cord injury 48 hours ago. What should the nurse assess for when monitoring for gastrointestinal complications? 1. A history of diarrhea 2. A flattened abdomen 3. Hyperactive bowel sounds 4. Hematest-positive nasogastric tube drainage

4. Hematest-positive nasogastric tube drainage Rationale: Development of a stress ulcer can occur after spinal cord injury and can be detected by Hematest-positive nasogastric tube aspirate or stool. The client is also at risk for paralytic ileus, which is characterized by the absence of bowel sounds and abdominal distention. A history of diarrhea is irrelevant.

The nurse is performing a neurological assessment on a client who had a stroke (brain attack). The nurse checks for proprioception using which assessment technique? 1. Tapping the Achilles tendon using the reflex hammer 2. Gently pricking the client's skin on the dorsum of the foot in 2 places 3. Firmly stroking the lateral sole of the foot and under the toes with a blunt instrument 4. Holding the sides of the client's great toe and, while moving it, asking what position it is in

4. Holding the sides of the client's great toe and, while moving it, asking what position it is in Rationale: A method of testing for proprioception is to hold the sides of the client's great toe and, while moving it, asking the client what position it is in. Tapping the Achilles tendon with a reflex hammer describes gastrocnemius muscle contraction. Pricking the skin on the dorsum of the foot in 2 different places describes 2-point discrimination. The plantar reflex is tested when the sole of the foot is stroked with a blunt instrument.

The nurse is creating a plan of care for a client with a diagnosis of stroke (brain attack) with anosognosia. To meet the needs of the client with this deficit, the nurse should include activities that will achieve which outcome? 1. Encourage communication. 2. Provide a consistent daily routine. 3. Promote adequate bowel elimination. 4. Increase the client's awareness of the affected side.

4. Increase the client's awareness of the affected side. Rationale: In anosognosia, the client exhibits neglect of the affected side of the body. The nurse will plan care activities that remind the client to perform actions that require looking at the affected arm or leg, as well as activities that will increase the client's awareness of the affected side. The remaining options are not associated with this deficit.

The home health nurse has been discussing interventions to prevent constipation in a client with multiple sclerosis. The nurse determines that the client is using the information most effectively if the client reports which action? 1. Drinking a total of 1000 mL/day 2. Giving herself an enema every morning before breakfast 3. Taking stool softeners daily and a glycerin suppository once a week 4. Initiating a bowel movement every other day, 45 minutes after the largest meal of the day

4. Initiating a bowel movement every other day, 45 minutes after the largest meal of the day Rationale: To manage constipation, the client should take in a high-fiber diet, bulk formers, and stool softeners. A fluid intake of 2000 mL/day is recommended. The client should initiate a bowel movement on an every-other-day basis and should sit on the toilet or commode. This should be done approximately 45 minutes after the largest meal of the day to take advantage of the gastrocolic reflex. A glycerin suppository, bisacodyl suppository, or digital stimulation may be used to initiate the process. Laxatives and enemas should be avoided whenever possible because they lead to dependence.

The nurse is creating a plan of care for a client with a stroke (brain attack) who has right homonymous hemianopsia. Which should the nurse include in the plan of care for the client? 1. Place an eye patch on the left eye. 2. Place personal articles on the client's right side. 3. Approach the client from the right field of vision. 4. Instruct the client to turn the head to scan the right visual field.

4. Instruct the client to turn the head to scan the right visual field. Rationale: Homonymous hemianopsia is a loss of half of the visual field. The nurse instructs the client to scan the environment and stands within the client's intact field of vision. The nurse should not patch the eye because the client does not have double vision. The client should have objects placed in the intact fields of vision, and the nurse should approach the client from the intact side.

The nurse is caring for a client with a diagnosis of right (nondominant) hemispheric stroke. The nurse notes that the client is alert and oriented to time and place. On the basis of these assessment findings, the nurse should make which interpretation? 1. Had a very mild stroke 2. Most likely suffered a transient ischemic attack 3. May have difficulty with language abilities only 4. Is likely to have perceptual and spatial disabilities

4. Is likely to have perceptual and spatial disabilities Rationale: The client with a right (nondominant) hemispheric stroke may be alert and oriented to time and place. These signs of apparent wellness often suggest that the client is less disabled than is the case. However, impulsivity and confusion in carrying out activities may be very real problems for these clients as a result of perceptual and spatial disabilities. The right hemisphere is considered specialized in sensory-perceptual and visual-spatial processing and awareness of body space. The left hemisphere is dominant for language abilities.

A postoperative craniotomy client who sustained a severe head injury is admitted to the neurological unit. What nursing intervention is necessary for this client? 1. Take and record vital signs every 4 to 8 hours. 2. Prophylactically hyperventilate during the first 24 hours. 3. Treat a central fever with the administration of antipyretic medications such as acetaminophen. 4. Keep the head of the bed elevated at least 30 degrees, and position the client to avoid extreme flexion or extension of the neck and head.

4. Keep the head of the bed elevated at least 30 degrees, and position the client to avoid extreme flexion or extension of the neck and head. Rationale: Avoiding extreme flexion and extension of the neck can enhance venous drainage and help prevent increased intracranial pressure. As a general rule, hyperventilation is avoided during the first 24 hours postoperatively because it may produce ischemia caused by cerebral vasoconstriction. Vital signs need to be taken and recorded at least every 1 to 2 hours. Central fevers caused by hypothalamic damage respond better to cooling (hypothermia blankets, sponge baths) than to the administration of antipyretic medications.

The nurse in the neurological unit is caring for a client with a supratentorial lesion. The nurse assesses which measurement as the most critical index of central nervous system (CNS) dysfunction? 1. Temperature 2. Blood pressure 3. Ability to speak 4. Level of consciousness

4. Level of consciousness Rationale: Level of consciousness is the most critical index of CNS dysfunction. Changes in level of consciousness can indicate clinical improvement or deterioration. Although blood pressure, temperature, and ability to speak may be components of the assessment, the client's level of consciousness is the most critical index of CNS dysfunction.

A client with a spinal cord injury expresses little interest in food and is very particular about the choice of meals that are actually eaten. How should the nurse interpret this information? 1. Anorexia is a sign of clinical depression, and a referral to a psychologist is needed. 2. The client has compulsive habits that should be ignored as long as they are not harmful. 3. The client probably has a naturally slow metabolism, and the decreased nutritional intake will not matter. 4. Meal choices represent an area of client control and should be encouraged as much as is nutritionally reasonable.

4. Meal choices represent an area of client control and should be encouraged as much as is nutritionally reasonable. Rationale: Depression frequently may be seen in the client with spinal cord injury and may be exhibited as a loss of appetite. However, the client should be allowed to choose the types of food eaten and when they are eaten as much as is feasible because it is one of the few areas of control that the client has left. There is no information in the question that would indicate that the client is anorexic or obsessive-compulsive or has a slow metabolism.

The nurse is caring for a client with an exacerbation of multiple sclerosis. Which medication(s) will the nurse expect to be prescribed to hasten recovery from the exacerbation? 1. Carbamazepine and phenytoin by mouth 2. Lioresal by mouth and diazepam intravenously 3. Phenytoin intravenously, then tapered to oral route 4. Methylprednisolone and cyclophosphamide intravenously

4. Methylprednisolone and cyclophosphamide intravenously Rationale: Intravenous methylprednisolone or adrenocorticotropic hormone in combination with cyclophosphamide may be prescribed to accelerate recovery from an exacerbation of multiple sclerosis. Carbamazepine may be prescribed for trigeminal neuralgia, and phenytoin may be prescribed to control seizures. Lioresal and diazepam are used to treat muscle spasticity.

The nurse is giving medication instructions to a client who is receiving phenytoin for epilepsy. Which instruction should the nurse include to promote adherence to the medication? 1. Discuss the self-limiting nature of epilepsy. 2. Explain how nonadherence does not account for treatment failure. 3. Involve one other person only in promoting adherence to prevent confusion. 4. Monitor plasma medication levels to provide information about compliance.

4. Monitor plasma medication levels to provide information about compliance. Rationale: Epilepsy is a chronic disease that requires regular and continuous therapy. It is not self-limiting, and nonadherence to the medication regimen results in treatment failure. Family and friends should be involved in the treatment regimen to help promote compliance. Monitoring plasma medication levels helps to provide information about adherence and can promote coaching and enhance compliance.

The nurse is providing instructions to a client beginning medication therapy with divalproex sodium for treatment of absence seizures. The nurse instructs the client that which represents the most frequent side or adverse effect of this medication? 1. Tinnitus 2. Irritability 3. Blue vision 4. Nausea and vomiting

4. Nausea and vomiting Rationale: The most frequent side and adverse effects of medication therapy with divalproex sodium are gastrointestinal (GI) disturbances, such as nausea, vomiting, and indigestion. The items in the other options are incorrect.

A client with a traumatic brain injury is on mechanical ventilation. The nurse promotes normal intracranial pressure (ICP) by ensuring that the client's arterial blood gas (ABG) results are within which ranges? 1. PaO2 60 to 100 mm Hg (60 to 100 mm Hg), PaCo2 25 to 30 mm Hg (25 to 30 mm Hg) 2. PaO2 60 to 100 mm Hg (60 to 100 mm Hg), PaCo2 30 to 35 mm Hg (30 to 35 mm Hg) 3. PaO2 80 to 100 mm Hg (80 to 100 mm Hg), PaCo2 25 to 30 mm Hg (25 to 30 mm Hg) 4. PaO2 80 to 100 mm Hg (80 to 100 mm Hg), PaCo2 35 to 38 mm Hg (35 to 38 mm Hg)

4. PaO2 80 to 100 mm Hg (80 to 100 mm Hg), PaCo2 35 to 38 mm Hg (35 to 38 mm Hg) Rationale: The goal is to maintain the partial pressure of arterial carbon dioxide (PaCo2) at 35 to 38 mm Hg (35 to 38 mm Hg). Carbon dioxide is a very potent vasodilator that can contribute to increases in ICP. The PaO2 is not allowed to fall below 80 mm Hg (80 mm Hg), to prevent cerebral vasodilation from hypoxemia, which can also result in an increase in ICP. Therefore, the remaining options are incorrect.

The nurse is caring for a client who is brought to the hospital emergency department with a spinal cord injury. The nurse minimizes the risk of compounding the injury by performing which action? 1. Keeping the client on a stretcher 2. Logrolling the client onto a soft mattress 3. Logrolling the client onto a firm mattress 4. Placing the client on a bed that provides spinal immobilization

4. Placing the client on a bed that provides spinal immobilization Rationale: Spinal immobilization is necessary after spinal cord injury to prevent further damage and insult to the spinal cord. Whenever possible, the client is placed on a special bed, such as a Stryker frame, which allows the nurse to turn the client to prevent complications of immobility while maintaining alignment of the spine. If a Stryker frame is not available, a firm mattress with a bed board under it should be used. The remaining options are incorrect and potentially harmful interventions.

The nurse has a prescription to begin aneurysm precautions for a client with a subarachnoid hemorrhage secondary to aneurysm rupture. The nurse would plan to incorporate which intervention in controlling the environment for this client? 1. Keep the window blinds open. 2. Turn on a small spotlight above the client's head. 3. Make sure the door to the room is open at all times. 4. Prohibit or limit the use of a radio or television and reading.

4. Prohibit or limit the use of a radio or television and reading. Rationale: Environmental stimuli are kept to a minimum with subarachnoid precautions to prevent or minimize increases in intracranial pressure. For this reason, lighting is reduced by closing window blinds and keeping the door to the client's room shut. Overhead lighting also is avoided for the same reason. The nurse prohibits television, radio, and reading unless this is so stressful for the client that it would be counterproductive. In that instance, minimal amounts of stimuli by these means are allowed with approval of the primary health care provider.

The client is admitted to the hospital with a diagnosis of Guillain-Barré syndrome. Which past medical history finding makes the client most at risk for this disease? 1. Meningitis or encephalitis during the last 5 years 2. Seizures or trauma to the brain within the last year 3. Back injury or trauma to the spinal cord during the last 2 years 4. Respiratory or gastrointestinal infection during the previous month

4. Respiratory or gastrointestinal infection during the previous month Rationale: Guillain-Barré syndrome is a clinical syndrome of unknown origin that involves cranial and peripheral nerves. Many clients report a history of respiratory or gastrointestinal infection in the 1 to 4 weeks before the onset of neurological deficits. On occasion, the syndrome can be triggered by vaccination or surgery.

The nurse is caring for a client with spinal cord injury (SCI) who is participating in a bowel retraining program. What should the nurse anticipate to promote during the bowel retraining program? 1. Sufficiently low water content in the stool 2. Low intestinal roughage that promotes easier digestion 3. Constriction of the anal sphincter based on voluntary control 4. Stimulation of the parasympathetic reflex center at the S1 to S4 level in the spinal cord

4. Stimulation of the parasympathetic reflex center at the S1 to S4 level in the spinal cord Rationale: The principal reflex center for defecation is located in the parasympathetic center at the S1 to S4 level of the spinal cord. This center is most active after the first meal of the day. Other factors that contribute to satisfactory stool passage are sufficient fluid and roughage in the diet and the Valsalva maneuver (which is lost with SCI). During defecation, the anal sphincter relaxes.

The nurse is creating a plan of care for a client with a diagnosis of stroke (brain attack). On reviewing the client's record, the nurse notes an assessment finding of anosognosia. The nursing care plan should address which manifestation related to this finding? 1. The client will be easily fatigued. 2. The client will have difficulty speaking. 3. The client will have difficulty swallowing. 4. The client will exhibit neglect of the affected side.

4. The client will exhibit neglect of the affected side. Rationale: In anosognosia, the client neglects the affected side of the body. The client either may ignore the presence of the affected side (often creating a safety hazard as a result of potential injuries) or may state that the involved arm or leg belongs to someone else. The remaining options are not associated with anosognosia.

A client with a probable minor head injury resulting from a motor vehicle crash is admitted to the hospital for observation. The nurse leaves the cervical collar applied to the client in place until when? 1. The family comes to visit. 2. The nurse needs to do physical care. 3. The primary health care provider makes rounds. 4. The results of spinal radiography are known.

4. The results of spinal radiography are known. Rationale: There is a significant association between cervical spine injury and head injury. For this reason, the nurse leaves any form of spinal immobilization in place until spinal radiographs rule out fracture or other damage. The remaining options are incorrect.

The nurse is caring for a client with trigeminal neuralgia (tic douloureux). The client asks for a snack and something to drink. The nurse should offer which best snack to the client? 1. Cocoa with honey and toast 2. Hot herbal tea with graham crackers 3. Iced coffee and peanut butter and crackers 4. Vanilla wafers and room-temperature water

4. Vanilla wafers and room-temperature water Rationale: Because mild tactile stimulation of the face can trigger pain in trigeminal neuralgia, the client needs to eat or drink lukewarm, nutritious foods that are soft and easy to chew. Extremes of temperature will cause trigeminal nerve pain. Therefore, the options that include cocoa, hot herbal tea, and iced coffee are incorrect.


संबंधित स्टडी सेट्स

Bonus Chapter 1: Concepts of Personality Development

View Set

Bio 110 - Chapter 6 (Cellular Respiration)

View Set